Download as pdf or txt
Download as pdf or txt
You are on page 1of 36

LEGALEDGE TEST SERIES

Part of the Most Comprehensive & Consistently Successful Study Material & Test Series Module, spanning across
both Physical and Online Programs in the entire Country. As a result, LegalEdge was able to engineer Clean-Sweep-
Landslide figures of a handsome 64 Selections under 100 ranks, and a whopping 273 selections under 500 ranks in
CLAT 2021. With AILET being no different, a total of 34 of our students found their way into NLU, Delhi in 2021.
In a nutshell, every second admit in a Top National Law School in 2021 came from the LegalEdge Preparation
Ecosystem.

MOCK COMMON LAW ADMISSION TEST 2022


MOCK CLAT #32

Scan this code after the test


Duration : 120 Minutes Candidate Name : _____________
Max. Marks : 150 Batch : _____________
Centre Name : __________ Contact No. : _____________

to punch in your answers


INSTRUCTIONS TO CANDIDATES (Test ID: 2479363)

1. No clarification on the question paper can be sought. Answer the questions as they are.
2. There are 150 multiple choice objective type questions.
3. There is negative marking of 0.25 for every incorrect answer. Each question carries ONE mark. Total marks are
150
4. You have to indicate the correct answer by darkening one of the four responses provided, with a BALL PEN
(BLUE OR BLACK) in the OMR Answer Sheet.
Example: For the question, "Where is the Taj Mahal located?", the correct answer is (b).
The student has to darken the corresponding circle as indicated below:
(a) Kolkata (b) Agra (c) Bhopal (d) Delhi
Right Method Wrong Methods

5. Answering the questions by any method other than the method indicated above shall be considered incorrect and
no marks will be awarded for the same.
6. More than one response to a question shall be counted as wrong.
7. Do not write anything on the OMR Answer Sheet other than the details required and, in the spaces, provided for.
8. You are not required to submit the OMR Answer Sheet and Test Paper after the test.
9. The use of any unfair means by any candidate shall result in the cancellation of his/her candidature.
10. Impersonation is an offence and the student, apart from disqualification, may have to face criminal prosecution.
11. You have to scan the QR code only after completion of offline test.
12. You cannot leave the examination hall without punching your answers on the portal.
SECTION-A : ENGLISH LANGUAGE

Directions(Q.1-Q.30): Read the following passage carefully and answer the questions that follow.
Passage (Q.1-Q.5): New high-speed cell phone services have raised concerns of interference with aircraft
operations, particularly as aircraft are landing at airports. The Federal Aviation Administration has assured
Americans that most commercial aircraft are safe, and AT&T and Verizon have agreed to hold off on installing
their new cell phone antennas near airports for six months. But the problem has not been entirely resolved.
Concerns began when the US government auctioned part of the C-band spectrum to wireless carriers in 2021 for
$81 billion. The carriers are using the C-band spectrum to provide 5G service at full speed, 10 times the speed
of 4G networks.
The C-band spectrum is close to the frequencies used by key electronics that aircraft rely on to land safely. Here
is why that can be a problem.
Wireless signals are carried by radio waves. The radio spectrum ranges from 3 hertz to 3,000 gigahertz and is
part of the electromagnetic spectrum. The portion of the radio spectrum that carries the signals from your phone
and other wireless devices is 20 kilohertz to 300 gigahertz.
If two wireless signals in the same area use the same frequency, you get garbled noise. You hear this when you
are midway between two radio stations using the same or similar frequency bands to send their information. The
signals get garbled and sometimes you hear one station, at other times the other, all mixed with a healthy dose
of noise.
Therefore, in the US, the use of these frequency bands is tightly regulated by the Federal Communications
Commission to ensure that radio stations, wireless carriers and other organisations are assigned “lanes”, or
frequency spectra, to use in an orderly fashion.
Modern aeroplanes use altimeters, which calculate the time it takes for a signal to bounce back from the ground
to determine a plane’s altitude. These altimeters are a vital part of automatic landing systems that are especially
useful in cases where there is low visibility.
So, if an altimeter interprets a signal from a wireless carrier as the rebounded signal from the ground, it may
think that the ground is closer than it is and prematurely try to lower the landing gear and do the other
manoeuvres that are needed to land an aircraft.
If interference with wireless carrier signals corrupts and garbles the altimeter’s radio signals, the altimeter may
not recognise the rebounded signal and thus be unable to figure out how close to the ground the plane is.
1. Which of the following can be inferred from the passage?
(a) AT&T and Verizon are mobile carriers who acquired their rights from the 81-billion-dollar Deal.
(b) AT&T and Verizon are federal Government agencies.
(c) The C-band spectrum is close to the frequencies used by key electronics that aircraft rely on to land safely.
(d) Most altimeters are barometric; it means they can measure altitude by calculating the location’s air pressure.
2. ‘You hear this when you are midway between two radio stations using the same or similar frequency bands to
send their information.’ Which part of speech has been used in the italicized word?
(a) Adjective (b) Adverb (c) Conjunction (d) Pronoun
3. Which of the following best reflects the title of the passage?
(a) The 4G signals are better than 5G ones
(b) 5G putting aircraft operation at risk.
(c) The pitfalls of 5G services.
(d) AT&T and Verizon Vs Federal Aviation Administration.
4. Select the meaning of the word ‘manoeuvres’, in the context of the passage
(a) A movement or series of moves requiring skill and care.
(b) A simulated military conflict carried out as an or exercise in personal development.
(c) A person who operates the flying controls of an aircraft.
(d) Two wireless signals in the same area using the same frequency create a garble noise.

Head Office: 127, Zone II, MP Nagar, Bhopal |+91-7676564400| https://www.toprankers.com Page 2 of 36
5. According to the passage which of the following is true?
(a) The cell phone towers interfere with the landing of airplanes.
(b) The cell phone towers interfere with the take-off of airplanes.
(c) The cell phone towers don’t interfere with the airlines industry.
(d) The cell phone towers are tuned to decrease their frequency when an aircraft is flying by.
Passage (Q.6-Q.10): In 1920, Bertrand Russell argued for a new social model that combined the advantages of
socialism and anarchism, and that basic income should be a vital component in that new society. More recently,
particularly after Covid highlighted large inequalities, and with rising concern that technology (particularly AI)
could both widen these inequalities and, perhaps, even change the nature of work, universal basic income (UBI)
has come under more serious consideration. It has already been tried to a limited extent in several countries.
We are now at a time when UBI must become an integral part of fiscal policy—not just because inequality is at
an all-time high, but also because, as we look forward, it seems clear that economic growth will likely be
sustainably lower than over the past 50 years, exacerbating the situation.
Economics is linked to the real world through the following identity: economic growth = population growth +
productivity growth.
Population growth has been slowing sharply in China and, to a lesser extent, in India; fertility rates are falling in
most of the developed world; and there is considerable anecdotal evidence showing that many young couples
are having just one child while many others (for a variety of reasons, interestingly including climate change) are
simply not having, or certainly not planning, babies. World Bank forecasts global population growth to fall from
1.03% this year to 0.5% in 2050 (and further to 0.03% in 2100). Thus, economic growth in the future will be
lower than in the past, unless there is game-changing productivity growth.
To be sure, several technologies unfolding will generate step increases in productivity—deeper penetration of
smart phones into lower income populations; smart homes becoming the norm; AI usage spreading across
industries; block chain technology making transactions more efficient, etc. On the other hand, over the past 50
years or so, productivity-enhancing technologies have entered the world, and become the norm by today—
personal computers, the internet and mobile phones are just three examples. Today, each of us is, on average,
already three times more productive than each individual was in 1961. While it is impossible to assess whether
productivity gains in the future will be higher, lower, or more or less the same as in the past, it is important to
note that gains from productivity appear, perhaps unsurprisingly, to taper off in time—the running average of
annual productivity gains has fallen from 3.47% in 1966 to 1.90% today.
https://www.financialexpress.com/opinion/universal-basic-income-ubis-time-has-come/2396494/
6. Determine the nature of the statement after carefully reading the passage.
Statement – Population has inverse relation to economic growth.
(a) Entirely False (b) Entirely True (c) Partially True (d) Partially False
7. Which of the following is not one the character that is influencing the current fall in the population rate?
(a) Declining fertility rates. (b) Climate change.
(c) Couples electing to have one or no child. (d) None of the above.
8. Identify the line in the passage that provides a normative assertion of the author’s theory?
(a) In 1920, Bertrand Russell argued for a new social model that combined the advantages of socialism and
anarchism.
(b) Today, each of us is, on average, already three times more productive than each individual was in 1961.
(c) Economics is linked to the real world through the following identity: economic growth = population
growth + productivity growth.
(d) We are now at a time when UBI must become an integral part of fiscal policy—not just because inequality
is at an all-time high, but also because, as we look forward, it seems clear that economic growth will likely
be sustainably lower than over the past 50 years, exacerbating the situation.
Head Office: 127, Zone II, MP Nagar, Bhopal |+91-7676564400| https://www.toprankers.com Page 3 of 36
9. ‘We are now at a time when UBI must become an integral part of fiscal policy—not just because inequality is at
an all-time high, but also because, as we look forward, it seems clear that economic growth will likely be
sustainably lower than over the past 50 years, exacerbating the situation.’
Give the antonym of the underlined word in the passage –
(a) Aggravating (b) Pacifying (c) Assaulting (d) Humiliating
10. Economic growth is likely to show which kind of trend in the coming years?
(a) Downward trend as population is declining gradually and productivity among individuals has decreased
over the years.
(b) Upward, as although both the elements of economic growth are not in sync, yet one outmatches the other’s
level.
(c) Downward, as population is on a declining slope, and there needs to be a drastic change in the productivity
growth to negate the effect of decreasing population.
(d) Upward as Universal Basic Income is going to be implemented all over the globe.
Passage (Q.11-Q.15): Every time I feel small-minded about my own insignificance in history, I turn to Jonathan
Swift’s Battle of Books.
Therein, you may recall, modern writers vent their spleen at the ancients for having “unduly” occupied the
heights of Parnassus, leaving them the lowly ridges.
The ancients of course make the simple argument that they have earned their places through deep thinking and
hard work, and that it were best for the moderns to do the same; rather than seek parity by cutting Parnassus
down to match the height of their own hillock, they had best work harder to rise to deserve the pinnacles where
the ancients rest.
I have always thought this is just the right talisman for those who are in a hurry to approximate the greatness of
others without quite earning it. And, worse, using what clout they have to erase that which rebukes their own
craven and insecure hubris.
How much better it would have been if the hymn that Bapu so adored, as do millions of all faiths across the
world adore, had been rather propagated more widely to enjoin respect for all faiths, if not for the Mahatma,
rather than evicted from the time-honoured melodies of Beating of the Retreat.
After all, it is hardly to be thought that erasing the hymn would ever succeed in either belittling the so exalted
and moving prayer or relegating Bapu from public memory.
As Shakespeare understood so well, often that which we seek to erase we perpetuate even more (the murder of
King Duncan in Macbeth being one case in point).
Here then was an opportunity lost to acquire a stake in the universal Gandhian legacy, and be counted among
the ancients.
Nor may the snuffing out of the Amar Jawan Jyoti succeed in the least to eject from collective memory and as
classical legend the scale and significance of the military victory of 1971, or enable a forgetting which might
help to foreground the “surgical strike” as a greater event.
So, whom Atal Bihari Vajpayee had christened Durga ought to have been left to occupy that pride of place,
rather than be amalgamated into a canny rearrangement with a modern flame dearer to the ruling heart.
The fact is any old honest Casper will continue to sing of that liberating battle, “It was a great victory” – the
official forgetting notwithstanding.
It is said that Netaji has been made to languish and needed to be rehabilitated. Never mind the hundreds of parks,
buildings, institutions, neighbourhoods etc. that bear his honoured name, the politically clouded coordinates
notwithstanding. And never mind also that the much-maligned Jawaharlal Nehru donned advocates’ garbs after
decades to appear in defence of the Indian National Army.
11. Which of the following literary devices has been used in the passage?
(a) Allegory (b) Allusion (c) Alliteration (d) Metaphors

Head Office: 127, Zone II, MP Nagar, Bhopal |+91-7676564400| https://www.toprankers.com Page 4 of 36
12. Select the meaning of the word ‘Hubris’
(a) Excessive pride or self-confidence
(b) Low pride or self-confidence
(c) Just the perfect amount of pride and self confidence
(d) No pride or self confidence
13. What does the author do when he/she feels small minded and insignificant?
(a) Tries to learn about Indian history
(b) Goes out dancing
(c) Turns to Jonathan Swift’s Battle of Books
(d) Remember the works of Jawaharlal Nehru.
14. Which of the following is true according to the passage
(a) If we try to eliminate something, it is very likely that we will perpetuate it instead
(b) If we try to eliminate something, we can do so successfully by perpetuating it instead
(c) Perpetuation of something means it's eradication from religion with optimism
(d) The ancients have earned their places through deep thinking and using survival skills.
15. ‘Therein, you may recall, modern writers vent their spleen at the ancients for having “unduly” occupied the
heights of Parnassus, leaving them the lowly ridges.’ What can be understood by the expression, vent their
spleens?
(a) To express gratitude (b) To express angry feelings
(c) To express discomfort (d) To express true feelings
Passage (Q.16-Q.20): “There is one case where a man has no right to ask at all for sex since they are not in a
marriage, but another case where he has a right sanctified with marital bond between them, he can expect
reasonable sexual relations with partner…we are not recognising the difference if we are saying that they are at
par.”
“If the legislature has thought that where parties are married and the only grievance of lady is that she was not
willing, if the legislature felt such a case should not be categorised as rape, is it something which is so
unconstitutional.”
“Can’t equate chalk and cheese.”
There is a clear thread running through the remarks made by Justice Shankar in the marital rape exception
hearings – how should a legitimate expectation of ‘reasonable sexual relations’ in a marriage affect how we are
to see, and punish, non-consensual sex within its bounds?
Senior advocates Rebecca John and Rajshekhar Rao have, in their responses, established that the right to sexual
autonomy must prevail over this ‘legitimate expectation’, whether or not it be a legal right. Justice Shankar
ostensibly agrees with this; he has clarified that his comments do not mean that non-consensual sex within
marriage should not be an offence at all. Yet, he continues to insist that one cannot “close one’s eyes” to the
qualitative difference between a marital and non-marital relationship.
To understand where Justice Shankar’s views seem to diverge, it might be appropriate to frame his inquiry more
narrowly. The absolute nature of a woman’s sexual autonomy and her right to say ‘no’ is, in fact, common ground
between the Bar and the Bench here (as it should be). Given that, Justice Shankar seems to be asking if the
legitimate expectations in a marriage make what is undoubtedly a violation of sexual autonomy a less severe or
a different violation from a violation outside marriage. As long as the law recognizes autonomy itself as an
absolute right, is it unconstitutional to punish some violations of this absolute autonomy as something other than
‘rape’; perhaps as the law does at present, where a wife that has been sexually assaulted by her husband can still
resort to filing a case for cruelty?
To summarise – is the violation of a victim’s sexual autonomy by her husband qualitatively different enough
from a violation by a stranger for it to be labelled differently or punished differently? The victim has an absolute
right to say no, but can we treat differently these two violations of that absolute right? This, at its core, is an
Article 14 issue of equal treatment.
Head Office: 127, Zone II, MP Nagar, Bhopal |+91-7676564400| https://www.toprankers.com Page 5 of 36
16. Select the meaning of the phrase ‘Chalk and cheese’
(a) If you describe two people or things as chalk and cheese, you mean that they are completely different from
each other.
(b) Similar from outside different from the inside
(c) Describing two different things or persons on some common ground.
(d) If you describe two people or things as chalk and cheese, you mean that one is superior to the other.
17. What has been said to be a common ground between the bar and the bench
(a) The courtroom (b) A woman’s sexual autonomy
(c) Marital rape (d) Article 14.
18. What does the author mean by the phrase ‘Bar and Bench’?
(a) A bar and a bench (b) The judges and the lawyers
(c) The stenographers who type judgments (d) The court room.
19. What is Justice Shankar’s main concern?
(a) Whether it’s alright to equate sexual relations without consent between two strangers to sexual relations
between a married couple where there is expectation of reasonable sexual relations.
(b) Whether it is alright to label sexual relations without consent in a marriage as ‘rape’.
(c) Whether the marital rape constitute as restricted to one gender only.
(d) Sexual autonomy is a constitutional right of a person, and violation of such must be met with serious
implications.
20. Which of the following can be inferred from the passage?
(a) Justice Shanker is in favour of regarding marital rape as heinous a crime as a non-consensual, coercive sexual
relations form a stranger.
(b) The marital bond between husband and wife does not allow for reasonable sexual relations with partner as
marital rape.
(c) Senior advocates Rebecca John and Rajshekhar Rao are in absolute congruence with Justice Shanker.
(d) The main concern regarding marital rape is whether the violation of a victim’s sexual autonomy by her
husband and violation by a stranger are the same in which the punishment meted out should be on the same
grounds?
Passage (Q.21-Q.25): There are not many places that I find it more agreeable to revisit when I am in an idle
mood, than some places to which I have never been. For, my acquaintance with those spots is of such long
standing, and has ripened into an intimacy of so affectionate a nature, that I take a particular interest in assuring
myself that they are unchanged.
I never was in Robinson Crusoe's Island, yet I frequently return there. The colony he established on it soon faded
away, and it is uninhabited by any descendants of the grave and courteous Spaniards, or of Will Atkins and the
other mutineers, and has relapsed into its original condition. Not a twig of its wicker houses remains, its goats
have long run wild again, its screaming parrots would darken the sun with a cloud of many flaming colours if a
gun were fired there, no face is ever reflected in the waters of the little creek which Friday swam across when
pursued by his two brother cannibals with sharpened stomachs. After comparing notes with other travellers who
have similarly revisited the Island and conscientiously inspected it, I have satisfied myself that it contains no
vestige of Mr. Atkins's domesticity or theology, though his track on the memorable evening of his landing to set
his captain ashore, when he was decoyed about and round about until it was dark, and his boat was stove, and
his strength and spirits failed him, is yet plainly to be traced. So is the hill-top on which Robinson was struck
dumb with joy when the reinstated captain pointed to the ship, riding within half a mile of the shore, that was to
bear him away, in the nine-and-twentieth year of his seclusion in that lonely place. So is the sandy beach on
which the memorable footstep was impressed, and where the savages hauled up their canoes when they came
ashore for those dreadful public dinners, which led to a dancing worse than speech-making. So is the cave where
the flaring eyes of the old goat made such a goblin appearance in the dark. So is the site of the hut where Robinson
Head Office: 127, Zone II, MP Nagar, Bhopal |+91-7676564400| https://www.toprankers.com Page 6 of 36
lived with the dog and the parrot and the cat, and where he endured those first agonies of solitude, which--strange
to say--never involved any ghostly fancies; a circumstance so very remarkable, that perhaps he left out something
in writing his record? Round hundreds of such objects, hidden in the dense tropical foliage, the tropical sea
breaks evermore; and over them the tropical sky, saving in the short rainy season, shines bright and cloudless.
21. It can be inferred from the passage that the author is
(a) A traveller (b) A wanderer (c) A bibliophile (d) An author
22. ‘For, my acquaintance with those spots is of such long standing, and has ripened into an intimacy of so
affectionate a nature, that I take a particular interest in assuring myself that they are unchanged.’ Which of the
following is the best interpretation of the given sentence?
(a) The author deeply associates with some of the places in the stories that he wishes them to be in their in situ
state.
(b) The author is extremely moved by the details of certain places that he wishes them to change so as it can be
praised by him only.
(c) The author has feeling of nostalgia for all the places that he has visited and re-visited.
(d) The author has developed deep affection for places that he has visited in his reverie that he wants them
unchanged.
23. According to the passage which of the following means the word ‘vestige’?
(a) Rubble (b) Vicissitude (c) Affirmation (d) Trace
24. ‘After comparing notes with other travellers who have similarly revisited the Island and conscientiously
inspected it, I have satisfied myself that it contains no vestige of Mr. Atkins's domesticity or theology, though
his track on the memorable evening of his landing to set his captain ashore, when he was decoyed about and
round about until it was dark, and his boat was stove, and his strength and spirits failed him, is yet plainly to be
traced.’ What can be understood by the given statement?
(a) The author, to satisfy his curiosity of wishing to know if such places exist, meets with other travellers to
reinforce his curiosity.
(b) The author, to satisfy his curiosity of wishing to know if such places exist, discusses with other avid readers
to reinforce his curiosity.
(c) The author, to satisfy his curiosity of wishing to know if such places exist, goads other avid readers to visit
those places to find out their authenticity.
(d) The author, to satisfy his curiosity of wishing to know if such places exist, reads the stories many times with
other avid readers to reinforce his curiosity.
25. According to the passage, what is the genre of the story books that the author reads?
(a) Fantasy (b) Fiction (c) Illusory (d) All of the above
Passage (Q.26-Q.30): As the day was pleasant, Madame Valmonde drove over to L'Abri to see Desiree and the
baby.
It made her laugh to think of Desiree with a baby. Why, it seemed but yesterday that Desiree was little more than
a baby herself; when Monsieur in riding through the gateway of Valmonde had found her lying asleep in the
shadow of the big stone pillar.
The little one awoke in his arms and began to cry for "Dada." That was as much as she could do or say. Some
people thought she might have strayed there of her own accord, for she was of the toddling age. The prevailing
belief was that she had been purposely left by a party of Texans, whose canvas-covered wagon, late in the day,
had crossed the ferry that Coton Mais kept, just below the plantation. In time Madame Valmonde abandoned
every speculation but the one that Desiree had been sent to her by a beneficent Providence to be the child of
her affection, seeing that she was without child. For the girl grew to be beautiful and gentle, affectionate and
sincere - the idol of Valmonde.

Head Office: 127, Zone II, MP Nagar, Bhopal |+91-7676564400| https://www.toprankers.com Page 7 of 36
It was no wonder, when she stood one day against the stone pillar in whose shadow she had lain asleep, eighteen
years before, that Armand Aubigny riding by and seeing her there, had fallen in love with her. That was the way
all the Aubignys fell in love, as if struck by a pistol shot. The wonder was that he had not loved her before; for
he had known her since his father brought him home from Paris, a boy of eight, after his mother died there. The
passion that awoke in him that day, when he saw her at the gate, swept along like an avalanche, or like a prairie
fire, or like anything that drives headlong over all obstacles.
Monsieur Valmonde grew practical and wanted things well considered: that is, the girl's obscure origin. Armand
looked into her eyes and did not care. He was reminded that she was nameless. What did it matter about a name
when he could give her one of the oldest and proudest in Louisiana? He ordered the corbeille from Paris, and
contained himself with what patience he could until it arrived; then they were married.
26. It can be inferred from the passage that
(a) Madame Valmonde was mother to Armand Aubigny.
(b) Madame Valmonde was grandmother to Desiree’s child.
(c) Madame Valmonde was grandmother to Desiree.
(d) Madame Valmonde was the owner of L'Abri.
27. Why, it seemed but yesterday that Desiree was little more than a baby herself; when Monsieur in riding through
the gateway of Valmonde had found her lying asleep in the shadow of the big stone pillar.
Which of the following can be inferred from the lines?
(a) Desiree was the biological daughter of Monsieur Valmonde by birth.
(b) Desiree was not the biological daughter of Monsieur Valmonde.
(c) Monsieur Valmonde found her daughter to be a wanderer by nature.
(d) Monsieur Valmonde was the caretaker of an orphanage.
28. Which of the following best expresses the meaning of the expression, ‘beneficent Providence’?
(a) The self-seeking Texans (b) The generous Mr. Valmonde
(c) The kind almighty (d) The benevolent Armand Aubigny
29. Which of the following is not supported by the passage?
(a) The toddler belonged to the party of Texans, who came in the area in a canvas-covered wagon.
(b) Madame Valmonde accepted the girl as a blessing of God for she had no child of hers.
(c) Desiree had recently become a mother to a baby.
(d) Armand Aubigny knew about Desiree’s past before tying nuptial knots.

30. Which of the following reflects the writing style of the author?
(a) Expository (b) Narrative (c) Analytical (d) Polemical

Head Office: 127, Zone II, MP Nagar, Bhopal |+91-7676564400| https://www.toprankers.com Page 8 of 36
SECTION-B : CURRENT AFFAIRS, INCLUDING GENERAL KNOWLEDGE

Directions (Q.31–Q.65): Read the information given below and answer the questions based on it.
Passage(Q.31-Q.35): The tension on the Russia-Ukraine border represents a major security crisis for the region,
with the potential to snowball into a broader conflict. Ukraine says that Russia has amassed around 90,000 troops
at the border, and US intelligence reports say that a Russian invasion of Ukraine is possible as early as next
month.

Russia has shown, as recently as 2014, that it is not averse to taking military action in Ukraine. In that year,
Russia seized [1] from Ukraine in what was the first time a European country annexed territory from another
country since World War Two.

Ukraine and Russia share hundreds of years of cultural, linguistic and familial links. As part of the Soviet Union,
Ukraine was the second-most powerful Soviet republic after Russia, and was crucial strategically, economically
and culturally. Ever since Ukraine split from the Soviet Union, both Russia and the West have vied for greater
influence in the country in order to keep the balance of power in the region in their favour.

31. Which of the following will replace [1] in the above passage?
(a) Georgia (b) Poland (c) Belarus (d) Crimea

32. Which of the following is the INCORRECT reason for the Russia-Ukraine Conflict?
(a) The Donbass region of eastern Ukraine has been facing a pro-Russian separatist movement since 2014.
(b) Russia seized Crimea from Ukraine in 2014.
(c) Ukraine has urged the Collective Security Treaty Organisation to speed up his country’s membership in the
alliance.
(d) Ever since Ukraine split from the Soviet Union, both Russia and the West have vied for greater influence in
the country.

33. ‘Minsk II’ peace agreement was signed by Russia and Ukraine in which of the following years?
(a) 2014 (b) 2015 (c) 2017 (d) 2020

34. Which of the following is true about NATO?


(a) North Atlantic Treaty Organization (NATO) is a military alliance established by the North Atlantic Treaty.
(b) Republic of North Macedonia was the last member joining NATO.
(c) There are a total of 30 members in NATO.
(d) All of the above

35. Ukraine became an independent country in which of the following years?


(a) 1989 (b) 1991 (c) 1993 (d) 1996

Passage(Q.36-Q.40): The Prohibition of Child Marriage (Amendment) Bill, 2021, which was introduced in Lok
Sabha on December 21, seeks to increase the minimum age of marriage of females to [1] years from the current
18 years.

“It is imperative to tackle gender inequality and gender discrimination and to put in place adequate measures to
secure health, welfare and empowerment of our women and girls and to ensure status and opportunity for them
at par with men,” said the Bill, which has been referred to the Parliamentary Standing Committee.

It is no secret that women lag behind men on almost all parameters, and many continue to be married even before
they reach the age of 18. The pandemic has aggravated many of these problems.

Head Office: 127, Zone II, MP Nagar, Bhopal |+91-7676564400| https://www.toprankers.com Page 9 of 36
The National Family Health Survey found that [2] per cent of women in the age group of 20 to 24 years were
married before they turned 18.

36. Which of the following will replace [1] in the above passage?
(a) 19 (b) 20 (c) 21 (d) 22

37. Which of the following will replace [2] in the above passage?
(a) 23.3 (b) 25.5 (c) 28.7 (d) 30.6

38. Which of the following statement is INCORRECT regarding marriage laws in India?
(a) In India, the minimum age of marriage was prescribed for the first time by the law known as the Sarda Act,
1929.
(b) Main reason given for raising the legal age of marriage for women by task force is to control the population.
(c) For Hindus, The Hindu Marriage Act, 1955 sets 18 years as the minimum age for the bride and 21 years as
the minimum age for the groom.
(d) In Islam, the marriage of a minor who has attained puberty is considered valid.

39. Which of the following International Convention, applicable in India prohibits Child Marriage?
(a) Convention on the Elimination of All Forms of Discrimination Against Women
(b) International Convention on the Rights of Children
(c) Universal Declaration of Human Rights
(d) Declaration of the Laws of Customs and Treaties.

40. The Committee led by who among the following suggested to raise the age of marriage for girls?
(a) Smriti Irani (b) Jaya Jaitley (c) Jaya Bachchan (d) Uma Bharti

Passage(Q.41-Q.45): Calling for justice for the 26/11 terror attacks in Mumbai (2008) and the Pathankot airbase
attack (2016) for the first time since the group was formed, Foreign Ministers of the Australia-India-Japan said
that the Quad is already cooperating on sharing intelligence on threats in the Indo-Pacific region.

The group of ministers, who held their fourth Quad ministerial meeting in [1] on Friday also resolved to speed
up delivery of more than a billion Covid vaccines to be manufactured in India, to hold a special meet on climate
change this year, and step efforts to ensure maritime security in the region. They announced plans for a Quad
summit including PM Modi, U.S. President Biden and Australian PM Morrison to be hosted by Japan’s PM
Kishida in Tokyo in the “first half of 2022”.

41. Which of the following will replace [1] in the above passage?
(a) Melbourne (b) Tokyo (c) Washington (d) New Delhi

42. Which of the following is NOT TRUE about QUAD?


(a) The idea of Quad was first mooted by Japanese Prime Minister Shinzo Abe in 2007.
(b) In November 2017, India, Japan, the US and Australia gave shape to the long-pending proposal of setting up
the Quad.
(c) In 2020, the trilateral India-US-Japan naval exercises expanded to include Australia, marking the first official
grouping of the Quad since its resurgence in 2017.
(d) None of the above

Head Office: 127, Zone II, MP Nagar, Bhopal |+91-7676564400| https://www.toprankers.com Page 10 of 36
43. China was one of the major topics of discussion in the First in-person QUAD leaders meet in September 2021.
Which of the following was discussed in the meet?
(a) The Line of Actual Control (LAC) was discussed by the QUAD leaders as one of several examples of
Chinese aggression.
(b) Issues related to Hong Kong, Xinjiang, Taiwan Strait and coercion of Australia, harassment around the
Senkaku, were also discussed.
(c) Concerns regarding Chinese cyberattacks on the USA targets (Microsoft Exchange and SolarWinds) and
also cybersecurity incidents in India, Japan and Australia were discussed.
(d) All of the above

44. All the QUAD countries participated in which of the following naval exercises in 2020?
(a) Malabar Exercise (b) Yudh Abhiyas
(c) Garuda Shakti (d) Nomadic Elephant

45. Two of the QUAD members, Australia and USA have announced a new grouping for Indo-Pacific, along with
which of the following countries?
(a) UK (b) France (c) Germany (d) Russia

Passage(Q.46-Q.50): Births in institutional facilities, such as a hospital, improved by nearly eight percentage
points but children who were either stunted or displayed signs of wasting only dropped by a maximum of three
percentage points, shows a comparison of National Family Health Survey-5 (NFHS-5) and NFHS-4.

The complete results of the NFHS-5 were made public on Wednesday. The NFHS-4 was released in 2014-15
and the latest, which captured population health indicators in [1] was delayed due to the pandemic.

India has also officially hit a total fertility rate (TFR) of [2] that indicates a decrease from the 2.2 in the NFHS-
4. According to the United Nations Population Division, a TFR of about 2.1 children per woman is called
replacement-level fertility. If replacement level fertility is sustained over a sufficiently long period, each
generation will exactly replace itself. The urban TFR is 1.6 and the rural TFR is 2.1.

46. Which of the following will replace [1] in the above passage?
(a) 2017-19 (b) 2016-18 (c) 2018-20 (d) 2019-21

47. Which of the following will replace [2] in the above passage?
(a) 1.9 (b) 2.0 (c) 2.1 (d) 2.3

48. Which of the following is TRUE about the National Family Health Survey?
(a) The First National Family Health Survey (NFHS-1) was conducted in 1992-93.
(b) It provides an indicator for tracking 30 Sustainable Development Goals (SDGs) that the country aims to
achieve by 2030.
(c) The funding for different rounds of NFHS has been provided by USAID, the Bill and Melinda Gates
Foundation, UNICEF, UNFPA, and Ministry of Health and Family Welfare, Government of India.
(d) All of the above

49. Who among the following is the Nodal Agency for the National Family Health Survey?
(a) International Institute for Population Sciences
(b) World Health Organisation
(c) Indian Organisation for Health Statistics
(d) National Statistical Organisation

Head Office: 127, Zone II, MP Nagar, Bhopal |+91-7676564400| https://www.toprankers.com Page 11 of 36
50. Which of the following is NOT TRUE about the National Family Health Survey 5?
(a) The most surprising reversals have happened in child stunting, which reflects chronic undernutrition.
(b) Infant Mortality Rate and Under 5 Mortality Rate data is mostly stagnant.
(c) There is an urban-rural gap as well as gender divide with respect to the use of the Internet in several states
and union territories.
(d) Child Wasting is reduced in all the states.
Passage(Q.51-Q.55): Manipur Chief Minister [1] on Sunday said the people of his state, including himself, want
the withdrawal of the AFSPA but only after mutual understanding with the Centre as national security is their
top priority.
"I believe AFSPA can be lifted gradually with the consent of the central government. But we must remember
there is no political stability in Myanmar and we share the border with that nation," the chief minister told PTI
in an interview here.
The first BJP chief minister of the poll-bound state also asserted that the elections will show a massive change
and his party will double the number of its seats.
The Bhartiya Janata Party managed to form a government in 2017 despite having just 21 seats compared to
Congress's 28 by joining hands with two local parties - NPP and NPF.
51. Which of the following will replace [1] in the above passage?
(a) N Biren Singh (b) Himanta Biswa Sarma
(c) Biplab Kumar Deb (d) Neiphiu Rio
52. Which of the following is TRUE about AFSPA?
(a) The British introduced such a legislation during Quit India Movement to Quell the protests.
(b) The present law effective in the Northeast was introduced in Parliament in 1958 by the then Home Minister,
G.B. Pant.
(c) The law first came into effect in 1958 to deal with the uprising in the Naga Hills, followed by the insurgency
in Assam.
(d) All of the above
53. Which of the following is the correct expansion of AFSPA?
(a) Army and Special Forces Powers Act
(b) Armed Forces Special Provisions Act
(c) Army and other Forces Special Provisions Act
(d) Armed Forces Special Powers Act
54. Which of the following committee has recommended to repeal the AFSPA?
(a) Justice Jeevan Reddy Committee (b) Santosh Hegde Committee
(c) Justice TSR Subramanyam Committee (d) Justice Dattu Committee
55. In which of the following years did the Supreme Court upheld the constitutionality of AFSPA?
(a) 1990 (b) 1998 (c) 1967 (d) 2002
Passage(Q.56-Q.60): Finance Minister Nirmala Sitharaman Tuesday presented the Union Budget 2022-23 in
Parliament. Here are the highlights from Sitharaman’s Budget speech, and an explanation and analysis of it.
In five big infrastructure projects, the government has proposed expanding highways in the country by 25,000
kilometres, allocating Rs 60,000 crore to the Nal se Jal scheme, five river link projects across various states, an
additional Rs 48,000 crore in the PM housing scheme, and boosting infrastructure development in the North
East.
The finance minister also announced the auction of 5G spectrum in 2022; proposed setting up 75 digital banking
units in 75 districts; announced a national programme for mental health, worsened by the Covid-19 pandemic;
and brought virtual currencies like cryptocurrency and non-fungible tokens under the tax net.
Head Office: 127, Zone II, MP Nagar, Bhopal |+91-7676564400| https://www.toprankers.com Page 12 of 36
56. Digital Currency announced in the Budget FY 2022-23 will be managed by which of the following?
(a) Reserve Bank of India (b) State Bank of India
(c) Ministry of Finance (d) All public sector banks can issue the Digital Currency
57. The digital currency proposed in the Budget 2022-23 will be taxed at the rate of which of the following?
(a) 15% (b) 20% (c) 25% (d) 30%
58. The FY Budget 2022-23 saw an increase in the Capital expenditure for the year 2022-23 by how much per cent?
(a) 25% (b) 28% (c) 32% (d) 35%
59. According to the Budget FY 2022-23, India’s economic growth has been estimated at which of the following?
(a) 7.5% (b) 8.9% (c) 9.2% (d) 9.8%
60. According to the Budget 2022-23, how many new generation Vande Bharat Trains to be manufactured during
the next three years?
(a) 200 (b) 250 (c) 300 (d) 400
Passage(Q.61-Q.65): The United Nations Educational, Scientific and Cultural Organization (UNESCO) has
picked up [1] among 49 cities as part of the creative city network under the Crafts and Folk Arts category.
“It is a proud moment for all of us. The process of nomination of [1] under the UNESCO ‘Creative City Network’
was undertaken and funded under the World Bank-funded Jhelum Tawi Flood Recovery Project. This is the
recognition of the historical crafts and arts of the city,” said Chief Executive Officer, Jhelum Tawi Flood
Recovery Project (JTFRP), Abid Rashid Shah.

The inclusion is likely to pave way for the city to represent its handicrafts on the global stage through UNESCO.

61. Which of the following will replace [1] in the above passage?
(a) Srinagar (b) Prayagraj (c) Varanasi (d) New Delhi

62. Which of the following is NOT TRUE about UNESCO?


(a) UNESCO is a specialized agency of the United Nations.
(b) UNESCO’s Headquarters are located in Geneva.
(c) It has 193 Members and 11 Associate Members.
(d) Three UNESCO member states are not UN members: Cook Islands, Niue, and Palestine.

63. Which of the following is TRUE about the UNESCO Creative Cities?
(a) It was created in 2004.
(b) The United Nations General Assembly has designated the 31st of October as World Cities Day.
(c) After the inclusion of this year, the total number of cities are 295.
(d) All of the above

64. Which of the following Indian Cities already has a title of Creative City in field of Crafts and Folk Art?
(a) Varanasi (b) Mumbai (c) Jaipur (d) Hyderabad

65. Which of the following Indian cities has a title of creative city in the field of gastronomy category?
(a) Chennai (b) Hyderabad (c) Mumbai (d) New Delhi

Head Office: 127, Zone II, MP Nagar, Bhopal |+91-7676564400| https://www.toprankers.com Page 13 of 36
SECTION – C: LEGAL REASONING

Directions (Q.66 – Q.105): Read the comprehensions carefully and answer the questions based on it.
Direction(Q.66-Q.70): Read the given passage carefully and answer the questions given below:
The Karnataka High Court has said that cricket match fixing does not amount to the offence of cheating and
therefore the offence under Section 420 IPC cannot be invoked against the alleged offenders.
Cheating is defined under Section 415 of the IPC as, if a person intentionally induces a person to do or omit to
do any act which he would not have done if he was not deceived to do so and the act has caused harm to that
person in body, mind, reputation or property, then the person who fraudulently, dishonestly or intentionally
induced the other person is said to cheat. Any dishonest concealment of facts which can deceive a person to do
an act which he would not have done otherwise is also cheating within the meaning of this section.
As can be seen from the definition above, there are certain constituents of this offence. They are dealt with in
more detail as follows:
i. Acting dishonestly: Section 24 of the IPC defines the term 'acting dishonestly' as, ―when the doing of any
act or not doing of any act causes a wrongful gain of property to one person or a wrongful loss of property
to a person, the said act is done dishonestly.
ii. Property: Property has a much larger meaning. It does not only include money but other things also. These
other things are those which are measurable in terms of money. Moreover, the property should be in full-
fledged ownership of the person and he must have the complete right to enjoy its use.
iii. Fraudulently: According to section 25, a person is said to do a thing fraudulently if he does that thing with
intent to defraud/decept but not otherwise.
iv. Mens rea: Mens Rea is an intention or action to perform a crime. It has to be proved beyond any doubt that
the accused deliberately contributed to a crime.
Source: Extracted, edited and recreated from: https://www.livelaw.in/news-updates/karnataka-high-court-
cricket-match-fixing-section-420-ipc-cheating-fir-quashed-kpl-players-190026, blog.ipleaders.in,
legalserviceindia.com.

66. Raghu, a young and enthusiastic dancer wants to participate in a dance competition in his locality. He approaches
Rudra who he has heard of being in the theatre industry in some capacity. Rudra tells Raghu that he has performed
at various dance festivals across the country and that he has four recorded-dance moves to his name. Taking his
word to be true, Raghu pays Rudra a certain good amount in order to train him with different dance steps.
However, later, Raghu discovers that Rudra has only one recorded-dance video to his name and that he has only
performed in the eastern part of the country, and nowhere else. Can Rudra be held liable for cheating?
(a) No, as Rudra did not cause any damage to Raghu either in his body, mind, reputation or property.
(b) No, as Rudra did not have the requisite mens rea to perform the crime of cheating.
(c) Yes, as Rudra dishonestly concealed the facts, thereby cheating Raghu and Raghu paid him on the basis of
concealed facts.
(d) Yes, as Raghu later discovered that Rudra had lied to him.

67. Nabil and Nazrul are brothers and Nabil borrowed Nazrul's Gaming Laptop for a week. While the Gaming Laptop
was with Nabil, his friend Hasan told him that certain keys in the laptop need to be tightened properly and that
he can get it done from a shop he often visits. Nabil handed the Gaming Laptop to Hasan, with the assurance
that he would get it back in three days. However, Nabil later discovers that Hasan ran away with the Gaming
Laptop and his brother Nazrul mentioned that the laptop’s keys was perfectly working. Can Hasan be held liable
for cheating Nabil?
(a) Yes, as Hasan made Nabil give him the Gaming Laptop by deceiving him.
(b) Yes, as Hasan had the intention of stealing the Gaming Laptop from Nabil.
(c) No, as Hasan did not intend to defraud Nabil.
(d) No, as the Gaming Laptop did not belong to Nabil.

Head Office: 127, Zone II, MP Nagar, Bhopal |+91-7676564400| https://www.toprankers.com Page 14 of 36
68. Considering the facts of the previous question, Nazrul sought to claim the offence of cheating. Can Hasan be
held liable for cheating Nazrul ?
(a) Yes, as the Gaming Laptop belonged to Nazrul, only he can claim the offence of cheating.
(b) Yes, as it was Nazrul who was deprived of his property.
(c) No, as Hasan caused Nabil to depart with the Gaming Laptop by deceiving him.
(d) No, as Nazrul did not have any knowledge about these events.

69. Abdul and Zoya enter into a contract for the supply of handmade carry bags to be supplied by Abdul to Zoya,
on the receipt of which Zoya would pay an amount stipulated in the contract. On the day of delivery of the
handmade carry bags, Zoya doesn't receive any product but Abdul claims that he delivered the goods to Zoya's
brother, thereby performing his part of the contract. In reality, Abdul has not delivered the handmade carry bags.
When Zoya discovers this, she sues Abdul for the offence of cheating. Can Abdul be held liable for the same?
(a) No, as Abdul did perform his part of the contract, i.e., delivery of the handmade carry bags.
(b) No, as Abdul did not intend to cause any damage to Zoya.
(c) Yes, as Abdul deceptively led Zoya to believe that he performed his part of the contract.
(d) Both the options A and B are correctly extracting the rationale of this reasoning.

70. Following the facts of the previous question, Abdul makes the delivery of the handmade carry bags on time to
Zoya. However, as the handmade carry bags had some defects, Zoya refused to make payment. Can Zoya be
held liable for cheating Abdul?
(a) Yes, as Zoya deceptively led Abdul to believe that she would pay the amount upon delivery of the product.
(b) Yes, as Zoya intentionally wanted to deprive Abdul the amount upon delivery of the carry bags, as per the
contract.
(c) No, as no ingredients of cheating are met.
(d) No, as the product had defects, Zoya was not bound to pay Abdul the amount stipulated under the contract.

Direction(Q.71-Q.75): Read the given passage carefully and answer the questions given below:
The Karnataka High Court dismissed writ petitions brought by Flipkart and Amazon collectively contesting the
Competition Commission of India order made under Section 26 (1) of the Competition Act, 2002. This case dealt
with the notion of res judicata and its applicability to CCI rulings.
The doctrine of Res Judicata has been defined in Section 11 of the Civil Procedure Code. It means the matter is
already judged which is that no court will have the power to try any fresh suit or issues which has been already
settled in the former suit between the same parties. Also, the court will not try the suits and issue between those
parties under whom the same parties are litigating under the same title and matter are already been judged and
decided by the competent court. When the court finds any suits or issues which has been already decided by the
court and there is no appeal pending before in any court, the court has the power to dispose of the case by granting
a decree of Res Judicata. This doctrine is based on the premises that if the matter is already decided by the
competent court, then no one has rights to reopen it with the subsequent suit unless new facts are discovered. It
also enacts the conclusiveness of the judgments as to the points decided, in every subsequent suit between the
same parties. The doctrine of Res Judicata is applied by the court where issues directly and substantially involved
between the same parties in the former and present suit, are same. For e.g., it may be that in former suit only part
of the property was involved whereas in present or subsequent suit whole property of the parties is involved.
Then court will grant a decree of Res Judicata.
Source: Amazon V. CCI: Analysis Of Whether Res Judicata Stays Applicable On Orders Passed By CCI
(livelaw.in)

Head Office: 127, Zone II, MP Nagar, Bhopal |+91-7676564400| https://www.toprankers.com Page 15 of 36
71. M was a sub-inspector and was dismissed from the service of D.I.G. he challenged the order of dismissal by
filing a writ petition in the High Court. He said that he did not get a reasonable opportunity of being heard before
the passing of the order. However, the argument was negatived and the petition was dismissed. He again filed a
petition on the ground that he was appointed by the I.G.P. and he had no power to dismiss him from service. The
defendant argued that the suit was barred by constructive res judicata. Based on your understanding of the
passage and the given facts, decide whether the defendant will succeed in his argument or not.
(a) The defendant will succeed as the plea was well within the knowledge of the plaintiff and M could have
taken this argument in his earlier suit.
(b) The defendant will not succeed as the contention made was different from the one in the earlier suit.
(c) The defendant will succeed as the plea was baseless and had no constructive reasoning to it.
(d) The defendant will not succeed as the plea was filed in a different petition all together.
72. Ramesh was the plaintiff who was injured while driving a vehicle on a public highway in the Darby region. Due
to a transit company that was occupying the street, the steering wheel of the machine operating was pulled by
the driver’s hand. This resulted in injury to the complainant. A suit was filed against the street railway in the
Court to recover damages. It was proved that negligence was there on the part of both the parties also known as
contributory negligence. The judgment was passed in favour of the defendant. Later action was again brought
against the same defendant based on the same cause of action and against the same transit company. The
judgment in the first proceeding was brought to the attention of the court. The plaintiff admitted that he was the
same person who was the plaintiff in the action brought earlier in court. Decide:
(a) The court will decide in favour of the defendant because of the judgment of previous case.
(b) The court will dismiss the suit on the grounds of the doctrine of res-judicata.
(c) The court will administer the suit and critically hear all the parties before coming to a conclusion.
(d) The court will decide in the favour of the plaintiff because it was a contributory negligence.
73. Ram filed a civil suit, a contention regarding the arbitration of the Court was taken by Vir. The objection was
sustained and the plaint was returned to the plaintiff for the presentation. The Revenue Court was incompetent
when Ram approached the Revenue Court so he returned the petition. Once again Ram filed a suit in the Civil
Court. Vir contended that the suit was barred by the doctrine of res judicata. Will Vir’s contention succeed?
(a) No, Vir’s contention will not succeed because the plaint was returned in the first instance where the suit was
filed.
(b) No, Vir’s contention will not succeed because of the simple reason that Ram approached the different courts.
(c) No, Vir’s contention will not succeed because the same suit was taken to revenue court.
(d) None of the above.
74. A suit was filed in the Court so that certain temples are called public temples. A similar suit was dismissed by
the Court two years ago and the plaintiff contended that it was negligence on the part of the plaintiffs (of the
previous suit) and therefore the doctrine of res judicata cannot be applied. Decide:
(a) The suit shall be administered because of different litigating parties.
(b) The suit shall be disposed off due to the applicability of doctrine of res-judicata.
(c) The facts are insufficient to decide.
(d) The suit shall be decided similar to that of previous suit.

75. Amit challenged the validity of an order of assessment under Article 226. The petition was dismissed on the
basis of merits. The Supreme Court also dismissed the appeal that was made against the order on the basis of
merits. Amit again filed another writ petition in the same High Court against the same order of assessment. This
time the petition was dismissed by the High Court. The Supreme Court held that the petition was barred by the
principle of res judicata. Which of the essentials of the principle of res-judicata are fulfilled here?
(a) Barred by the Supreme Court of India.
(b) Dismissal of petition.
(c) Both option A and B are correct regarding the said principle.
(d) None of the above are correct regarding the said principle.
Head Office: 127, Zone II, MP Nagar, Bhopal |+91-7676564400| https://www.toprankers.com Page 16 of 36
Passage(Q.76-Q.81): Recovery of weapon used for the commission of crime is not a sine qua non (essential
condition) for convicting an accused, the Delhi High Court recently reaffirmed.
The judge noted that the argument that the weapon of offence in a case of attempt to murder/culpable homicide
charges was not recovered has no merit as the testimonies of the complainant and his brother were cogent and
consistent.
Given the nature and extent of injuries on the victim in the instant case, the single-judge observed that the
ingredients of the offence punishable under Section 307 IPC i.e. attempt to murder should be proved against
the appellant beyond any shadow of doubt. "In view of the fact that the complainant was medically examined
within 45 minutes of the incident, the contention bears no merit and is rejected," the Court held.
"As observed and held by this Court in catena of decisions nobody can enter into the mind of the accused and
his intention has to be ascertained from the weapon used, part of the body chosen for assault and the nature of
the injury caused. Considering the case on hand on the aforesaid principles, when the deadly weapon - dagger
has been used, there was a stab injury on the stomach and near the chest which can be said to be on the vital
part of the body and the nature of injuries caused, it is rightly held that the appellants have committed the
offence under Section 307 IPC i.e. attempt to murder.”
The Court took note of the pre-existing enmity between the appellant and the complainant, the fact that the
complainant had suffered two injuries during the incident, and observed that the nature of the injury was grievous
indicating that the appellant had the requisite intention and knowledge that such injuries could have been fatal.
In IPC it has also been mentioned that where death is caused by bodily injury, the person who causes such bodily
injury shall be deemed to have caused the death, although by resorting to proper remedies and skilful treatment
the death might have been prevented. Though person may use reasonable force to protect himself under self-
defence if required in such situation.
Source: https://www.barandbench.com/news/recovery-of-weapon-used-for-crime-not-essential-for-conviction-
delhi-high-court

76. A was running after H with a machete in hand with murderous intent and had finally cornered him in an alley
and was about to commit the offence. However, as A struck the blow, H dodged and ran from under him but not
before being gashed open in the shoulder. This led to H running to a hospital, by which time he had lost a lot of
blood and was on the verge of passing out. However, even after arduous attempts of the hospital authorities, H
could not be saved. Would A be liable for murder here?
(a) Yes, as H died due to the shoulder wound caused due to A’s actions.
(b) No, as H died due to excessive blood loss and not the wound primarily.
(c) No, as H succumbed to the injury much later and not via the act.
(d) Yes, as A had the intent, injurious weapon and wound caused by A enough to hold him liable for murder.

77. In the above case, had A had the intent to murder H, and had he cornered H in order to do so, but had wrongly
dealt the injury to B’s neck, who was standing next to H, would A be liable for murder here?
(a) No, as he did not intend to murder B
(b) Yes, as his actions caused B his life.
(c) No, as the nature of the injury is not grave and murder cannot be discernibly charged here.
(d) Yes, as A had murderous intent which took B’s life and shall be duly charged for the same.

78. Saya was murdered on the Meerut highway and his corpse was not found for the next two days. Once recovered,
it was sent for examination and the report returned incomplete findings due to passage of the 2 days. However,
it was found that Maya had purchased degradable and quick dissolving poison, which was with him that night.
Later it was found that Saya had duped Maya out of 2 lakh rupees in the previous month and Maya was bitter at
the same. Can Maya be discernibly convicted of murder here?
(a) No, as there is incomplete evidence of there been any poisoning here.
(b) No, as the report has not mentioned anything with regards to the poison.
(c) Yes, as there is a clear chain of events that can be drawn from the facts.
(d) Yes, as Maya clearly had the intent, the means and the opportunity to murder Saya.
Head Office: 127, Zone II, MP Nagar, Bhopal |+91-7676564400| https://www.toprankers.com Page 17 of 36
79. Sumit, a 17 year old boy was being tried for allegedly having murdered his classmate, Arvind, on account of
Arvind having died in the bathroom when Sumit was there. Guard has seen sumit coming out of washroom with
stains over shirt. Upon the body being found by the security, a small compass was also found near the body and
as per medical reports a large wound in the spine of Arvind was discerned as the reason for him bleeding out to
death. While considering these facts at hand, the court also noted that while the compass belonged to Sumit, no
more evidence for his conviction could be collected. Can the court hold Sumit as guilty here?
(a) No, as there is no intent on his part to be convicted.
(b) No, as no evidence pertaining to his conviction has been noted to be present here.
(c) Yes, as the evidence is clearly present in the form of the compass and the spine-wound.
(d) Yes, as the evidence points toward mal-intent and act on the part of Sumit.

80. Subhash was being chased by a loan shark for recovery of the money that he had borrowed from the shark but
had defaulted on. While being chased, the loan shark decided to lure him into running into an isolated
construction park, where Subhash, while running, fell over. To his dismay, Subhash started reaching for the
shovel nearby in order to attack the goon and in hastiness in self-defence, he absently grabbed the pickaxe and
threw it at the loan shark. This resulted in him being badly injured and succumbing to the injuries after being
rushed to the hospital. Can Subhash be held liable for murder here?
(a) No, as the intent behind throwing the tool was self-defense and not murder.
(b) Yes, as the action of throwing a deadly weapon at a person is tantamount to murder.
(c) Yes, as the action of self-defense has not been defined in law.
(d) No, as the loan shark finally died due to bleeding out and not due to the tool being thrown.

81. In the above case, had Subhash not thrown a pickaxe, but had merely thrown a lathi at his leg without knowing
that he had osteoporosis (weak bones), resulting in him suffering severe fractures leading to the loan shark
stumbling and then falling into a 20 feet deep pit, resulting in his death, would then Subhash had been liable for
murder?
(a) No, as the loan shark died due to his own fault, i.e, having the intent to murder.
(b) No, as the loan shark was suffering from a disease and died due to the same.
(c) No, as the contents of the offence of murder have not been met yet.
(d) No, as the intent behind throwing the lathi was not merely faze the aggressor and not incapacitate him
completely.

Passage(Q.82-Q.85): Section 311 CrPC empowers the court to summon a material witness, or to examine a
person present at “any stage” of “any enquiry”, or “trial”, or “any other proceedings” under CrPC, or to summon
any person as a witness, or to recall and re-examine any person who has already been examined if his evidence
appears to it, to be essential to the arrival of a just decision of the case. Undoubtedly, CrPC has conferred a very
wide discretionary power upon the court in this respect, but such a discretion is to be exercised judiciously and
not arbitrarily. The power of the court in this context is very wide, and in exercise of the same, it may summon
any person as a witness at any stage of the trial, or other proceedings. The court is competent to exercise such
power even suo motu if no such application has been filed by either of the parties. However, the court must
satisfy itself, that it was in fact essential to examine such a witness, or to recall him for further examination in
order to arrive at a just decision of the case.

Kumar Ohri, J., while discussing the scope of Section 311 of the Criminal Procedure Code, 1973 with regard to
recalling and cross-examining the witness expressed that,it is the duty of every Court to ensure that fair and
proper opportunities are granted to the accused for just decision of the case.
A petition was filed under Section 482 of the Criminal Procedure Code on behalf of the petitioner seeking setting
aside the orders of the lower court whereby the opportunity of the petitioner to cross-examine the witness (PW-
1) was closed and an application filed under Section 311 CrPC seeking recall of the said witness was dismissed.
(SOURCE : Fair trial is the hallmark of criminal procedure, it entails not only rights of victims but also interest
of accused: Delhi HC, livelaw)
Head Office: 127, Zone II, MP Nagar, Bhopal |+91-7676564400| https://www.toprankers.com Page 18 of 36
82. Choose an alternative which best describes the context of the passage:
I. Power under section 311 must be exercised, provided that the evidence that is likely to be tendered by a
witness, is germane to the issue involved.
II. The power conferred under Section 311 CrPC must therefore, be invoked by the court only in order to meet
the ends of justice, for strong and valid reasons, and the same must be exercised with great caution and
circumspection.
III. The very use of words such as “any court”, “at any stage”, or “or any enquiry, trial or other proceedings”,
“any person” and “any such person” clearly spells out that the provisions of this section have been expressed
in the widest possible terms, and do not limit the discretion of the court in any way.
IV. Adducing of evidence by the accused in support of his defense is not a valuable right and allowing the same
will be against the interest of justice.
(a) I, II & III (b) I & II (c) II &IV (d) Only I

83. Chandu filed a petition in HC under section 311 of Crpc alleging that he was denied presenting an alibi for his
defense an application filed under Section 311 CrPC seeking recall of the said witness was dismissed by the
lower court. Decide
(a) The lower court's order should be overturned since it was made in the accused's best interests.
(b) Lower court’s order will be upheld as interest of accused do not have find any place in bear reading of section
311.
(c) The lower court's order will be upheld as if the accused's discretion was an act of arbitrariness.
(d) The lower court's judgment should be reversed on the grounds that it was necessary to examine such a
witness, or to recall him for further examination, in order to arrive at a just decision in the case.

84. In the light of passage given, choose a correct option:


(a) Section 311 of the Code of Criminal Procedure, 1973 is confined to Court witnesses.
(b) Section 311 of the Code of Criminal Procedure, 1973 does not apply to defense witnesses.
(c) Section 311 of the Code of Criminal Procedure, 1973 cannot be exercised after the accused had closed his
defense.
(d) Section 311 of the Code of Criminal Procedure, 1973 does include “accused” as witness

85. The accused had been tried for the offence of attempt to murder under section 307 of IPC. It was the case of the
prosecution that accused was at home with his wife and mother-in-law when an altercation occurred. It
was alleged that Suraj returned intoxicated, went to the kitchen, grabbed the axe, and whacked the mother-in-
law and wife, causing grievous injury to the mother-in-law and simple injury to the wife. The prosecution failed
to offer testimony from the two witnesses who drove the two to the hospital, which would have been crucial in
reaching a fair decision in the case. The prosecution filed a motion under section 311 of the CrPC asking the
court to summon the two witnesses after the trial was completed. Decide
(a) The plea will be dismissed as section 311 can be invoked at any stage of the trial but not after it completion.
(b) The plea will be entertained as it is in fact essential to examine such a witness, or to recall him for further
examination in order to arrive at a just decision of the case.
(c) The plea will be entertained as the court has such power even suo motu if no such application has been filed
by either of the parties if it satisfied that such testimonies are essential to arrive at a just decision of the case.
(d) It will be the discretion of the court and in exercise of the same, it may summon any person as a witness at
any stage of the trial, or other proceedings.

Head Office: 127, Zone II, MP Nagar, Bhopal |+91-7676564400| https://www.toprankers.com Page 19 of 36
Passage(Q.86-Q.90): Subramanian Prasad, J., while explaining the facts of application for bail, cancellation of
bail and rejection of an application for bail, made an observation that, “Personal liberty is one of the cherished
constitutional freedoms. Once granted to an accused pending completion of the Trial, it must only be retracted
in the face of grave and exacerbating circumstances.”
When can a Court seize the liberty of an accused under trial?
Stating that a Court must tread with the utmost circumspection, and only after an in-depth examination of the
situation and new emergent facts and on finding supervening circumstances and overwhelming evidence that the
accused has been abusing the liberty granted to him by the Court, Bench explained when a Court can exercise
its jurisdiction in seizing the liberty of an accused under trial.
Supreme Court in the decisions of Delhi Admn. V. Sanjay Gandhi, (1978) 2 SCC 411 and Dolat Ram v. State of
Haryana, (1995) 1 SCC 349, expounded the position in law vis a vis cancellation of bail.
The power conferred under Section 439(2) [cancellation of bail] CrPC has to be exercised in a discreet fashion,
without dwelling on the merits of whether bail should have been granted or not and only upon viewing the
subsequent conduct of an accused. The power is coupled with the reserve and caution, akin to the usage of the
High Court’s inherent powers given under Section 482 CrPC.
Application for Cancellation of Bail and Grant of bail are different from each other, Bench added that High Court
will not exercise its jurisdiction to interfere with an order of bail granted by Special Judge if there is no serious
infirmity in it.
In the present matter, Court found the order of the ASJ to be well reasoned requiring no interference. Lastly, the
Court dismissed the petition noting that Court has not made any observation on the nature/manner of
investigation, and if an application challenging the nature/manner of investigation is filed by the complainant,
the Trial Court is requested to consider the same. [Charu Soneja v. State (NCT of Delhi), 2022 SCC OnLine Del
5, decided on 3-1-2022]
(Source: Cancellation of Bail v. Rejection of Bail: Delhi HC explains when a Court can seize liberty of an
accused undertrial, SCCONLINE BLOGPOST)

86. Complainant filed a case against her husband, mother-in-law, and brother-in-law for non-bailable offences under
Sections 354, 354A, 354B, 406, 498A, 506, 509, and 34 IPC. Respondents filed an anticipatory bail plea under
Section 438 CrPC, anticipating arrest. Can the respondents be granted bail?
(a) No, the respondents cannot be given bail because they committed a non-bailable offence.
(b) They can seek anticipatory bail since personal liberty is one of the most treasured constitutional liberties.
(c) No, it can only be given after a thorough investigation and after completion of inquiry by the trial court.
(d) Yes, as accused can be granted bail pending completion of the Trial.

87. Continuing the similar facts as above, suppose in an appeal under section 439 of CrPC, Judge found on the
grievance of the complainant, that the matter had not been investigated fairly or that the investigating agencies
acted in connivance with the accused and submitted the report in favor of accused. Decide
(a) The court can cancel the bail at any time on finding that the accused had been abusing the liberty granted to
him by the Court.
(b) The court has the authority to seize an accuser’s liberty while he is on trial if there is overwhelming evidence
that the accused has abused the liberty granted to him by the Court.
(c) The court cannot seize the liberty of an accused under trial once granted.
(d) If there are no exacerbating circumstances, the court cannot revoke the personal liberty.

Head Office: 127, Zone II, MP Nagar, Bhopal |+91-7676564400| https://www.toprankers.com Page 20 of 36
88. Pick a statement that offers an anomaly of the passage's context:
I. In an event of considering an application for cancellation of bail of an accused under Section 439(2) CrPC,
it has to be done by the HC by not only dwelling on the merits of whether bail should have been granted or
not but also upon viewing the subsequent conduct of an accused.
II. only In the event of grave and exacerbating circumstances, the court may revoke the liberty granted to an
accused undertrial.
III. The court lacks jurisdiction under Section 439(2) CrPC to interfere with a Special Judge's bail order if there
is a substantial flaw in it.
IV. The power granted by Section 439(2) CrPC must be used discreetly.
(a) II & IV (b) I &IV (c) III & II (d) I & III

89. Assertion: in an application filed under section 439 (2) of CrPC, Court found the order of the ASJ to be well
reasoned requiring no interference and hence uphold the decision of ASJ.
Reason: The Court will exercise its jurisdiction to interfere with an order of bail granted by Special Judge if there
is serious infirmity in it.
(a) Both A and R are true and R is correct explanation of A.
(b) Both A and R are true but R is not correct explanation of A
(c) A is true but R is false.
(d) A is false but R is true.

90. Madhu, who is dissatisfied with the special judge's order granting bail to the accused in the rape case, files an
application with the HC under section 439(2) of the CrPC, claiming that the investigation authorities failed to
conduct a proper investigation and that the accused bribed the officials in order for the report to be generated in
favor of the accused. Thereby seeking cancellation of liberty granted to accused pending completion of trial.
Decide
(a) The court will seize the liberty of an accused under trial.
(b) The court can dismiss the petition as an application challenging the nature/manner of investigation is filed
by the complainant.
(c) The court cannot dismiss the petition as court cannot retract personal liberty of an accused except in cases
of grave and exacerbating circumstances.
(d) The court will dismiss the petition and order the trial court to consider the issue raised by Madhu.

Passage(Q.91-Q.95): The Supreme Court on Wednesday began hearing arguments on the legal question of
whether an adult male member other than an aggrieved woman, has a right to file an application under Section
21 of the Protection of Women from Domestic Violence Act, for seeking visitation rights with respect to a child.
Referring to Section 21 of the Domestic Violence Act, which allows an aggrieved person to make an application
for grant temporary custody of the child, Attorney General submitted that, the problem is that as far as an
aggrieved person is concerned, it is only a woman. A man cannot be an aggrieved person under the DV Act, he
said. To which court denied.
The question is, can a husband come directly to HC and invoke parens patriae jurisdiction on basis that
otherwise he will have to go under various other Acts and the High Court has inherent jurisdiction so it could
then decide upon interim custody until wife or husband goes to court and applies for permanent custody. Parens
patriae would be the jurisdiction if he move under the Guardians and Wards Act, Special Marriage Act or the
Hindu Marriage Act, which have separate sections for custody." Court affirmed the same.
Section 21 Domestic Violence Act. Custody orders.—Notwithstanding anything contained in any other law for
the time being in force, the Magistrate may, at any stage of hearing of the application for protection order or
for any other relief under this Act grant temporary custody of any child or children to the aggrieved person or
the person making an application on her behalf and specify. If necessary, the arrangements for visit of such child
or children by the respondent can be made: Provided that if the Magistrate is of the opinion that any visit of the
respondent may be harmful to the interests of the child or children, the Magistrate shall refuse to allow such
visit.
Head Office: 127, Zone II, MP Nagar, Bhopal |+91-7676564400| https://www.toprankers.com Page 21 of 36
(SOURCE: Not Allowing Husband To Seek Child's Custody Under Domestic Violence Act Leads To
Multiplicity Of Proceedings: Supreme Court Tells AG, Livelaw)
91. Dharmendra was recently found not guilty in a murder case. His wife left him because she believed that staying
with him would influence the child's upbringing and that he would grow up to be like his father. The youngster
is eight years old. Under Section 21 of the Protection of Women from Domestic Violence Act, Dharmendra filed
a plea at the high court. Choose a suitable alternative for the wife so that the court would give her custody and
deny Dharmendra any visitation rights.
(a) The mother will be granted custodial rights by the court since the law states that custody of a minor child
must always remain with the mother.
(b) If the court is satisfied that Dharmendra’s visit is not harmful to his child's interest, the court will grant
Dharmendra custodial rights.
(c) Taking into account the plaintiff's prior criminal record, the court will grant the mother with the custody
rights.
(d) Because section 21 of the Protection of Women from Domestic Violence Act grants temporary custody of
any child or children to the mother solely, the court will award the mother custodial rights.

92. Under what circumstances an aggrieved person other than the women, can be granted custodial rights?
(a) It will be the discretion of the court upon appreciating the evidence placed on records.
(b) Nobody other than the women is entitled to get custodial rights under section 21 of the Protection of Women
from Domestic Violence Act.
(c) An aggrieved person other than the women, can seek custodial rights through other acts if not provided under
section 21 of the Protection of Women from Domestic Violence Act.
(d) An aggrieved person other than the women, can be granted visitation under section 21 of the Protection of
Women from Domestic Violence Act.

93. Sudha and Radha were married under the provisions of a special marriage legislation as they contracted same
sex marriage. Radha had always desired a child, so the couple decided to use IVF technology, with Sudha
agreeing to bear the child in her womb. After a few years together, the couple decided to separate. Sudha held
custody of the kid since she was the one who carried it in her womb, not Radha. Sudha claims that she is the
biological mother, not Radha. Decide who, under section 21 of the Protection of Women from Domestic
Violence Act, holds custody powers in this case.
(a) Due to the fact that Section 21 is not gender-neutral provision, Sudha, the biological mother, will have
exclusive custody rights.
(b) Both Radha and Sudha will hold custodial rights since an aggrieved person, as mentioned in the section 21
language, can only be a woman.
(c) Because this is a unique case, the court's decision will be made at its discretion.
(d) Can’t be determined due to incomplete information.

94. Choose an appropriate misinterpretation made in context of the passage:


I. An aggrieved person under section 21 come directly to HC and invoke parens patriae jurisdiction.
II. On an application by the husband, the magistrate can grant him visitation rights to visit his children under
Section 21 of the Protection of Women from Domestic Violence Act.
III. Husband has no right to approach under domestic violence act for visitation, and for custody he has to move
a separate petition before civil court under separate act.
IV. Section 21 deals with the custody of any child or children to the aggrieved person or the person filing an
application on her behalf.
(a) I & IV (b) Only IV (c) III & IV (d) ONLY III

Head Office: 127, Zone II, MP Nagar, Bhopal |+91-7676564400| https://www.toprankers.com Page 22 of 36
95. In the present case, a case under the Domestic Violence Act was filed against the petitioner by his wife. On an
application by the husband, the Trial Court had granted him visitation rights to visit his children under Section
21 of the Protection of Women from Domestic Violence Act.
(a) Trial courts order will be set aside as the husband cannot file an application for visitation rights under section
21.
(b) The order of the court is valid as it is based upon the application made by the aggrieved person for making
an arrangement of visitation rights in favor of the husband.
(c) Trial courts order will be upheld by the HC if the husband invokes parens patriae jurisdiction before a HC.
(d) The trial’s court order will not sustain as the court does not have the jurisdiction to pass such an order.

Passage(Q.96-Q.100): The Calcutta High Court on Tuesday permanently injuncted four advertisements by the
makers of Baidyanath Chyawanprash Special for being disparaging to all other brands including Dabur
Chyawanprash [Dabur India Limited v. Shree Baidyanath Ayurved Bhawan Pvt Ltd].
While partially allowing the plea, the Court stated that the advertisement was based on false information and the
comparison was misleading to the consumers.
“These commercials have the potential to influence consumer’s purchase preference in the market and it also
harms its rivals, hence, they must be used with caution. There should be a balance between the right of
commercial speech and the interest of public and competitors,” the order recorded.
Baidyanath argued that it had a right to commercial speech as a part of freedom of speech and expression which
could not be curtailed; The Court, however, held that commercial speech was not protected in serious negative
comparisons.
Essentially, it was derived from these judgments that the Court must apply the reasonable man test when deciding
an issue of disparagement.
The question to be examined must be whether a serious comparison was made or not, since a comparison of
“Better or Best” based on truthful claims was permitted. Further, the impact of the advertisement must be
examined as to whether it gives the impression that a rival product is defective.
It was stated that only “puff” and honest comparison was allowed and even generic disparagement was
objectionable. Finally, it was noted that comparative advertising campaign should be ‘comparison positive’,
promoting healthy competition.
(Source: Chyawanprash ads: Calcutta High Court injuncts Baidyanath Ayurved from disparaging Dabur
trademark, bar and bench )

96. Maggi was banned by India's food regulator in 2015 after tests revealed it contained excessive lead and was
mislabelled with the flavour enhancer MSG. After a judge lifted the ban after compliance were made with food
regulation, the product was reintroduced into markets. In a commercial for its competitor, chings Chinese, they
displayed a package that looked precisely like Maggie's and compared the two products in terms of the
ingredients utilized by the former and stated that how their product is healthier in terms of ingredients used. Can
Chings Chinese be held liable for influencing consumer’s purchase preferences in the market and it also harming
the goodwill of its rivals?
(a) The advertisement was based on false information and hence misleading.
(b) The advertisement was based on truth and hence the comparison made is valid.
(c) In their defence, the Chings Chinese might invoke the right to commercial speech as part of their freedom of
speech and expression.
(d) The advertisement was making a negative comparison hence misleading.

Head Office: 127, Zone II, MP Nagar, Bhopal |+91-7676564400| https://www.toprankers.com Page 23 of 36
97. Continuing with the facts above, consider if Chings Chinese advertised how their product is everything a
consumer needs, outpacing all other brands in terms of the healthy ingredients utilized in their products which
impacted the buying pattern. Determine chings chinise's liabilities.
(a) They will be liable for making a negative comparison between the two products.
(b) They will be liable for influencing consumers' buying choices in the market, as well as undermining
competitors' goodwill.
(c) They are not liable because they have a right to commercial speech as part of their right to freedom of speech
and expression, which cannot be curtailed.
(d) They will not be held accountable on application of reasonable man test in deciding liability on a case of
disparagement

98. Jumbo Glasses unveiled a new product line that incorporates a new technology that makes the glasses dust-
resistant. Where other rivals lack originality, it was demonstrated in their ad. Is this considered a healthy form
of competition?
(a) It will pass the test of reasonableness and hence the ad is promoting healthy competition.
(b) It will not pass the test of reasonableness and hence not promote healthy competition.
(c) The comparison was made on truthful claims and hence the ad is promoting healthy competition.
(d) The comparison made is a negative comparison and hence the ad is not promoting healthy competition.

99. Choose a sentence that does not accurately reflects the passage's context:
(a) It's wrong to make a sneering remark.
(b) Positive comparison is protected under the right to commercial speech.
(c) Comparison made on false claims cannot take defence of right to commercial speech.
(d) The concepts of commercial speech and public and competitive interests are not mutually exclusive.

100. Choose a statement that does not explains the application of test of reasonableness when deciding an issue of
disparagement:
(a) Whether the comparison made is positive or negative.
(b) When the comparison made is not of a serious nature.
(c) Whether the comparison offered has the ability to impact a customer's market purchasing selection.
(d) When there is a conflict between the freedom of commercial speech and the public's and competitors'
interests.

Passage(Q.101-Q.106): The Delhi High Court has reiterated that the occurrence of cause of action at a place is
a determining factor both under sec. 20 of the Code of Civil Procedure and sec. 134 of the Trade Marks Act to
attract the jurisdiction of the Court at such place.
A single judge bench comprising of Justice Manoj Kumar Ohri observed thus:
"If under Section 20 CPC, jurisdiction of the Court is attracted by virtue of location of the defendant’s place of
business or from where the defendant is carrying on its business or working for gain, under Section 134 of the
Trade Marks Act, it is the plaintiff’s office location or from where he is carrying on business, is a material factor"
Furthermore, it said:
"The occurrence of cause of action or any part thereof, at a place, is held to be a determining factor, both under
Section 20 CPC, and Section 134 of the Trade Marks Act, to attract jurisdiction of the court at such place."
Observing that the defendant's products were available for sale and delivery in Delhi through third party
marketplaces, the Court opined thus:
"it is prima facie established that the Defendant’s offending products are not only freely sold on amazon.in, but
are also available for sale to customers in Delhi on other third party marketplace websites e.g., Amazon, Flipkart,
Snapdeal, Indiamart and Shopclues, which are universally accessible, including to customers in Delhi."
Furthermore, it was observed that: "..if some part of cause of action has arisen at a place where the plaintiff has
its branch/subordinate office, Courts at that place will have jurisdiction to entertain a suit against infringement
and passing off."
Head Office: 127, Zone II, MP Nagar, Bhopal |+91-7676564400| https://www.toprankers.com Page 24 of 36
(SOURCE : Occurrence Of Cause Of Action Is A Determining Factor For Attracting Jurisdiction Of Courts
Under CPC, Trademarks Act: Delhi High Court Reiterates, LIVELAW)

101. The Court was hearing an application in a suit brought by V Guard Industries (plaintiff) seeking a permanent
injunction to prevent N Guard (defendant) from infringing and passing off its trade mark and design. The
defendant filed the application, contesting the High Court's competence to hear the suit due to a lack of territorial
jurisdiction, and requesting that the plaint be dismissed. Questioning the jurisdiction of the Delhi Court, it was
the defendant's case that it was not carrying the business in Delhi and that none of the plaintiff or defendant had
their registered office in Delhi. Decide the grounds on which the plaint can be dismissed
(a) The place of cause of action is outside the territorial jurisdiction of the court.
(b) The cause of action has been arisen at a place where the plaintiff has its branch/subordinate office.
(c) The place of cause of action is at third party marketplaces.
(d) The place of cause of action in the matters of infringement and passing off of trade mark has been a vexed
legal issue.

102. Continuing with the preceding facts, suppose the plaintiff alleges that the Defendant's products are available on
third-party marketplace websites such as Amazon, Flipkart, and Indiamart, which can be accessed within
the Court's territorial jurisdiction, and that the offending products were delivered in Delhi. Decide
(a) The court has the jurisdiction to try the case as the cause of action arises in Delhi.
(b) The court does not have the jurisdiction to try the case as a third party marketplace website like Amazon is
not adequate to attract the jurisdiction of the Court.
(c) The court has the jurisdiction to try the case as the cause of action has arisen at a place where the plaintiff
carries his business or works for gain.
(d) The court lacks jurisdiction to hear the case since the defendant's place of business does not attract present
court's jurisdiction under Section 20 CPC.

103. Dharni, a Bhopal resident, signed a deal with "foodlite goods marketing Ltd." to distribute her food products
throughout Delhi and Mathura. Which court will have jurisdiction to try the issue if there is a dispute between
the parties, according to section 20 of the CPC?
(a) Dharni’s place of business i.e., from Bhopal.
(b) Where the plaintiff is carrying on its business or working for gain i.e., Delhi and Mathura.
(c) Defendant’s place of business or where defendant is carrying on its business or working for gain i.e., Delhi
and Mathura.
(d) Defendant’s place of business or where defendant is carrying on its business or working for gain for the
plaintiff.

104. Choose the statement which provides an anomaly of the context of the passage:
I. Section 20 clearly provides that a Court within whose local limits the cause of action, “wholly or in part”,
arises, would have territorial jurisdiction to try the suit.
II. Section 20 deals with the institution of suits in the Code of Civil Procedure.
III. Objection as to place of suing shall be allowed in the court of first instance is the essence of section 20.
IV. The occurrence of cause of action or any part thereof, at a place, is held to be a determining factor, to attract
jurisdiction of the court at such place.
(a) II & IV (b) II & III (c) I & IV (d) II

105. Chandu lives in City A, while his partner lives in City B. The location of their business is C, which is located in
the middle of the cities of A and B. Which court will have jurisdiction?
(a) The place where chandu resides.
(b) The place where the cause of action arise.
(c) At C, the place of business.
(d) The courts of Cities A, B, and C are all competent to hear the case.
Head Office: 127, Zone II, MP Nagar, Bhopal |+91-7676564400| https://www.toprankers.com Page 25 of 36
SECTION - D: LOGICAL REASONING

Passage(Q.106-Q.110): Much interest has been generated by the Budget’s statement that RBI will issue digital
rupees during 2022-23. It’s a reiteration of a 2017 suggestion by GoI’s high-level inter-ministerial committee
that asked RBI to introduce its own central bank digital currency (CBDC). A CBDC is a digital form of paper
currency. Holders of CBDC can transfer money digitally instead of handing over currency notes. Globally, pilot
projects on CBDC have been underway since 2014. However, progress is slow because this seemingly simple
innovation can have unforeseen consequences.

India today is one of the 50 odd monetary jurisdictions with a digital retail payment system that runs 24/7 and
transfers are near real time. RBI’s pilot survey on retail payments done in six cities over December 2018 and
January 2019 showed that digital payments were catching on fast. While cash was the preferred mode of payment
for transactions up to Rs 500, digital payments were more popular for higher amounts. As digital payments have
been growing at over 50% over the last five years, the trend is clear. From a user’s standpoint, there is no real
incentive to switch to a CBDC as a growing proportion of retail transactions are already done digitally.

For RBI, however, a CBDC is not just another digital payment option. It has implications for the banking system
that are not entirely clear. If there is a partial shift away from bank deposits to CBDCs, what are the implications
for the role of banks in credit creation? By extension, what is the implication for monetary policy? The
disintermediation of banks by the central bank is an area that is being closely scrutinised by the world’s leading
central banks. One of them, Sweden’s Riksbank, launched its e-krona project in 2017. It’s still studying both its
need and potential impact on Sweden’s economy.

The case here isn’t against the introduction of a CBDC. Rather, it is about the need to be flexible about deadlines
in this endeavour. CBDCs may be inevitable but the need for RBI to cover for all the possible risks is non-
negotiable. The timeline should be flexible.

106. What is the best representation of the central idea of the passage?
(a) Considering the wider implications of CBDC, the RBI should drop the idea.
(b) Taking time over CBDC is important for RBI, considering the uncertainty.
(c) The RBI has the responsibility of making workable plans for CBDC.
(d) Introducing CBDC may have more stark consequences than what is expected.

107. The passage supports which of the following inferences?


(a) The increase in digital payments would pose challenges for CBDC.
(b) The general public would not hesitate to shift to CBDC from digital payments.
(c) Competing with digital retail payment is the biggest challenge for CBDC.
(d) Role of banks in credit creation would decrease after introduction of CBDC.

108. With which of the following forms of introduction of CBDC would the author agree?
(a) Going full force to implement CBDC.
(b) Phased implementation of CBDC
(c) Boycott other digital payments in order to introduce CBDC.
(d) Delay CBDC till systems are fool-proof.

109. With which of the following statements would the author not agree with?
i. CBDC would play a significant role in reducing money laundering.
ii. Cryptocurrency has increased the popularity of CBDCs.
iii. The idea of introducing CBDC in the recent budget is novel.
(a) All of the above (b) Only I and II
(c) Only II and III (d) Either I or II and III
Head Office: 127, Zone II, MP Nagar, Bhopal |+91-7676564400| https://www.toprankers.com Page 26 of 36
110. Out of the following paragraphs, which paragraph has played the role of summarising the arguments of the
author?
(a) The first paragraph (b) The second paragraph
(c) The third paragraph (d) The fourth paragraph

Passage(Q.111-Q.114): US President Joe Biden finds himself in a particularly galling situation regarding his
administration’s China policy. Because it’s not just his political foes who are panning his handling of a rampant
Beijing; the searing criticism is coming from the simpatico.

Perhaps the perils of giving into the left cabal in the Democratic Party have come to bite Mr. Biden in the fleshy
parts. For, he is being accused by pro-establishment policy makers, sections of the media, and party lawmakers
of effectively following what has been derisively described as a ‘Trump Lite’ approach towards China that
continues, according to them, to be destructive and unimaginative.

The tough line on China being followed by Mr Biden, thankfully sans the bombast of his predecessor, is justified
by his officials as one that has strong bi-partisan support. According to foreign policy expert Jeffrey A. Bader,
when asked to distinguish its China strategy from that of its predecessor’s, the Biden Administration says it
favours a multilateral approach of rallying allies, in contrast to the unilateralism of the America First
practitioners.

Scholars such as Mr Bader quote opinion polls to assert that Democrats do not have a visceral dislike of China
which Republicans do and argue that in taking an uncompromising stand, President Biden is guilty of continuing
to pursue a Trump-era policy under the guise of bi-partisanship at the expense of grassroots Democratic views.
What these critics from within fail to recognise is that politicians usually have their ear to the ground and
whatever opinion polls may claim, Mr Biden is attempting to keep the Democrats from lurching to the left when
it comes to China.

On the substantive issues, he has done well to jettison the incendiary and provocative language of his predecessor
while continuing to insist that meaningful cooperation, as opposed to transactional economic and trade ties, with
Beijing would be possible only if there are fundamental changes in its hegemonic approach and undemocratic
state apparatus.

Further, by recognising and calling out the Chinese desire for domination and gearing up international diplomacy
including strengthening the Quad, launching AUKUS, doubling down on its East Asia security alliance, and
keeping its strategic focus on the Asia-Pacific to ensure the Communist Party of China doesn’t have a free run,
Washington feels it is on the right track.

The strategy of “decoupling” from China by continuing a high tariff regime, validating the FBI’s sharp focus on
Chinese and Chinese-Americans conducting research at American universities especially in sensitive dual-use
technologies, and closing down various exchange programmes including with the Centre for Disease Control
given the data-fudging by the Chinese which was blatant during the coronavirus pandemic, are apt too. Crucially,
President Biden has indicated that acceptance of the One-China Policy is not sacrosanct given the aggressive
moves by Beijing against Taiwan. Trump Lite or not, this is a policy approach that needs to be supported by all
sensible sections of the American establishment.

111. What is the opinion of the author about the administration of Mr. Biden?
(a) Mr. Biden’s administration on China should be supported.
(b) Mr. Biden has a very similar stance on China as his predecessor.
(c) The take of Mr. Biden on China is starkly different from Trump.
(d) Mr. Biden’s administration is conservative, to say the least.

Head Office: 127, Zone II, MP Nagar, Bhopal |+91-7676564400| https://www.toprankers.com Page 27 of 36
112. If the information given in this passage is true, then which of the following must be true as well?
(a) Simpatico is a term that is used to describe one’s competitors.
(b) The author does not feel aligned with Trump’s provocative language.
(c) Mr. Biden has plans to work with China on some major fronts.
(d) China would have to be sympathetic for its hegemony to prevail.

113. With which of the following options would the author agree with?
(a) Jefferey Bader is a wolf disguised as a sheep in Biden’s administration.
(b) Mr. Biden considers working with allies more suitable than working alone.
(c) Mr. Biden has to pay back America’s allies for the deeds of Mr. Trump.
(d) Democrats have an instinctive disinclination towards China.

114. What is the role played by Mr. Bader’s statement in the context of the passage?
(a) It is one of the premises which strengthens the author’s arguments.
(b) It is a statement which describes the evidence which the passage refutes.
(c) It provides information that directly contradicts the author’s main argument.
(d) Mr. Bader’s statement is well taken by the author and it is his main conclusion.

Passage(Q.115-Q.119): Start-ups in India continue to proliferate in megacities. The flexibility to operate from
remote locations and away from cities being the fulcrum for start-ups, it is strange to note that small towns still
get nudged away by their big brothers. As per the recently unveiled Economic Survey of 2021–22, Delhi has
overtaken Bengaluru to become the start-up capital of India. The Survey tells us that start-ups in the country
have grown remarkably over the last six years, thanks to policy support by the government. With over 14,000
being recognised in 2021–22, their total number in the country has gone up to 61,400, as of January 10, making
India the third largest start-up ecosystem in the world after the US and China.

What is more exciting is the fact that a record 44 start-ups have achieved unicorn status in 2021, taking the
overall tally to 83. India overtook the UK to emerge as the third-highest country in number of unicorns after the
US and China. As of January 14, India’s 83 unicorns have reported a total valuation of $278 billion. That is
indeed unparalleled growth.

As per a recent Nasscom-Zinnov report, India will have 200 unicorns by 2025, and the ecosystem will account
for 37,000 tech start-ups. The emergence of the ecosystem has been imperative in providing direct and indirect
job opportunities, accounting for 6.6 lakh direct jobs and more than 34 lakh indirect jobs. Among sectors,
financial services and banking, edu-tech and supply chain management have been at the forefront of creating
most of the employment opportunities.

The need of the hour is to encourage a pan-India growth of the ecosystem. The government had started a system
of rankings to encourage states to provide a conducive environment. Last year, Gujarat took the best performer
ranking, while Karnataka and Kerala got the top performers’ titles. During the pandemic, there is a certain
accumulation of new start-ups in megacities. The policies by the government should aim at letting start-ups
bloom in smaller towns so that they can provide huge employment opportunities across the country.

115. It can be inferred from the passage that:


(a) The government has done something favourable for the start-ups of India.
(b) It is comparatively easier to build a start-up Unicorn in the US than in India,
(c) A rankings system to evaluate states on Start-ups is not the way forward.
(d) Start-ups are the way forward to reduce employment in the world.

Head Office: 127, Zone II, MP Nagar, Bhopal |+91-7676564400| https://www.toprankers.com Page 28 of 36
116. According to the author, which of the following would qualify as a logical course of action?
(a) An initiative that provides the start-ups from rural areas a seed money.
(b) Big Corporates funding the start-ups and contributing to build a start-up ecosystem.
(c) A policy that mandates the start-ups to employ 10 individuals from day one.
(d) A completely government funded start-ups in smaller towns.

117. Which among the following can be concluded from the passage?
i. The number of start-ups in Bengaluru has reduced.
ii. The growth rate of new start-ups being registered in Bengaluru has dipped.
iii. A large number of start-ups are being formed in Bengaluru, but are not registered.
(a) Only i and ii (b) Only i and iii
(c) None of the above. (d) All of the above.

118. The following statements either represent a fact or an opinion. Find the odd one out.
(a) The flexibility to operate from remote locations and away from cities being the fulcrum for start-ups, it is
strange to note that small towns still get nudged away by their big brothers.
(b) With over 14,000 being recognised in 2021–22, their total number in the country has gone up to 61,400, as
of January 10, making India the third largest start-up ecosystem in the world after the US and China.
(c) What is more exciting is the fact that a record 44 start-ups have achieved unicorn status in 2021, taking the
overall tally to 83. India overtook the UK to emerge as the third-highest country in number of unicorns after
the US and China.
(d) The need of the hour is to encourage a pan-India growth of the ecosystem. The government had started a
system of rankings to encourage states to provide a conducive environment. Last year, Gujarat took the best
performer ranking, while Karnataka and Kerala got the top performers’ titles. During the pandemic, there is
a certain accumulation of new start-ups in megacities.

119. Which of the following structures best describes the structure of the last paragraph of the passage?
(a) A possible solution to the problem is presented which is subsequently rejected due to its limited application.
(b) A few legitimate questions are raised to the solutions presented in the previous paragraphs.
(c) A solution to the problem is presented with a few examples and a brief explanation about how the solution
could help.
(d) A possible solution is portrayed and then its applicability is held in most of the cases.

Passage(Q.120-Q.124): The Supreme Court’s order quashing the one-year suspension of 12 BJP legislators by
the Maharashtra Assembly last year has corrected an unjustifiable excess of the Assembly. It will hopefully serve
as a guideline for other Assemblies and Parliament in the exercise of their powers to deal with disorderly conduct
in the House. Legislatures are increasingly resorting to the blunt weapon of suspensions to keep Opposition
members out of the House for longer periods and for even flimsier, and even invented reasons. The Chhattisgarh
Assembly suspended 11 BJP members last month. During the winter session of Parliament, 12 members of the
Rajya Sabha were suspended on grounds that were not convincing, and for their alleged actions during the
previous session. It is always members of the Opposition who face such action. The rules and procedures for
dealing with disorderly conduct are not followed in many cases of suspension.

The court held that the suspension of the Maharashtra MLAs was “substantively illegal and irrational”. It said
that suspensions should be limited to the session in which the alleged disorderly conduct takes place, and should
not be carried over to the next. This is because the suspension of a legislator is not a punishment but a measure
of self-protection of the legislature intended to ensure orderly conduct of its proceedings. It rightly pointed out
that the response to disruption should actually be a graded one, starting with the naming of the recalcitrant
legislator and going on to more serious action. The court felt rightly that the suspension of a legislator for a
whole year is worse than expulsion of that member. In the case of expulsion, a by-election would be held within
six months so that the constituency is represented in the House.
Head Office: 127, Zone II, MP Nagar, Bhopal |+91-7676564400| https://www.toprankers.com Page 29 of 36
The court also spelt out the dangers of the use of such power by governments. It said that a government with a
thin majority “could manipulate the numbers of the Opposition party in the House in an undemocratic manner’’
by suspending members and governments could pass legislation by keeping away the hassle of Opposition
members out of the House. This was, in fact, the criticism made against the suspension of 12 Rajya Sabha MPs
during the winter session. The court also noted that “the Opposition will not be able to effectively participate in
the discussions or debates in the House owing to the fear of members being suspended’’. That is why the court
thought that such suspensions are a danger to democracy. The court’s order should make legislatures realise that
there are limits to their power and that they should exercise it wisely and democratically.

120. The information presented in the passage supports which of the following inferences?
(a) The opposition members are at a weaker position than those in power as far as getting suspended from the
legislature is concerned.
(b) The Chattisgarh Assembly is more proactive in suspending its legislators than the Assembly of Maharashtra.
(c) The suspension of large numbers of Members of Parliament is within the parameters of democratic conducts.
(d) The court is unjustifiably intervening in the matters concerning the legislative assembly.

121. Which of the following has been used by the Supreme Court as a premise to support its order?
i. The suspension of the BJP MLAs in Chhattisgarh was not rational.
ii. Suspending legislators undemocratically could ease passing of a legislation.
iii. The main intention of suspension of legislators is not punishment.
(a) Only i and ii (b) Only ii and iii
(c) Only i and iii (d) All of the above

122. Which of the following best represents the central argument of the passage?
(a) The Supreme Court’s order is a reminder to the legislatures to use its powers wisely and democratically.
(b) The irrational suspension of the Maharashtra MLAs by the legislature is not healthy for democracy.
(c) The Court should take note of the action taken by the legislatures and quash any rule that is unparliamentary.
(d) The Supreme Court serves a reminder that the House should work within constitutional parameters.

123. With which of the following statements would the author of the passage disagree with?
I. The members of a legislature from the ruling party are at more risk of getting suspended from the house.
II. Members of the legislature who create disruption in the House should be dealt with in an acute manner.
III. It is better to suspend an MLA rather than permanently dismissing him/her from the legislature.
(a) Only I (b) Only III. (c) I and III (d) I, II and III

124. Based on the passage, read the following statements and choose the correct option.
I. Suspensions should be limited to the session where the disruption was caused.
II. The suspension of MLAs has been made in an undemocratic manner.
(a) Both I and II are positions held by the author.
(b) Both I and II are correct and II is an explanation for I.
(c) Both I and II are correct and I is an explanation for II.
(d) I is correct and II is incorrect.

Passage(Q.125-Q.129): In the background of a raging pandemic, the hope from the Budget 2021-22 was that
the urban centres, which saw the worst kind of forced-reverse migration in recent times, would find some redress.
The budget speech began with the recognition of the crisis of the pandemic but the finance minister, whilst
recognizing the role played by essential workers, offered very little to them. The budget aims to “strategically
disinvest”, promote “infrastructure development” and address the electoral challenges in the upcoming state
elections. The pandemic and the resultant economic crisis for marginal and worker groups - many of whom are
in the urban areas - get a rare mention at best.

Head Office: 127, Zone II, MP Nagar, Bhopal |+91-7676564400| https://www.toprankers.com Page 30 of 36
The budget can be analysed under three categories of urban schemes. First, the ones being pushed through
aggressively, like the thrust on infrastructure augmentation in urban centres. Under the guise of “raising the share
of public transport in urban areas” the budget pushes through unsustainable debt-ridden expansion of metro rail
network. The city bus service systems, wherever operating, will be converted into “innovative PPP models” to
enable private sector players to enter bus services in the cities. This will further dismantle the existing largely
public bus transport in favour of metro rail. This is further augmented by giving the car industry a “voluntary
vehicle scrapping policy” and not pushing through a strong policy for electric vehicles and non-motorised
transport. The increase in vehicles and the consequent rise in air pollution is expected. But the outlay promises
only a Rs 2,217 crore programme for air pollution - a mere palliative for cities that are some of the most polluted
centres in the world.
The second set of policies are the ones that are continued, but without any infusion of funds. There are hardly
any new or increased allocations in key sectors as housing, sanitation and services. The PMAY scheme has been
extended by one more year. Similar continued allocations are visible for AMRUT and Swachch Bharat. Though
there are continued allocations, there was no mention of the much-showcased smart cities initiative. The budget
speech is silent on it as it has become one of the biggest embarrassments due to lack of implementation and
policy failure.
The third set of measures are the ones where there is a grievous absence of any substantial allocations, only
rhetoric and promises are made in the budget. The best examples are of migrant housing. Instead of government
providing immediate thrust on housing and labour hostels, they will be given tax concessions, and there are no
separate allocations that ensure the delivery of housing options to migrant workers whose suffering during the
pandemic will remain in our collective memory. There is also a section on labour welfare in the budget, but it
fails to delineate any measures and resources to the lost livelihoods, reduced wages, and income insecurity that
the workers face currently. The budget does mention of universal social security and online portals to access
information on workers but gives little cash to the resource-starved informal sector that runs the urban centres.
An urban employment guarantee scheme was the most urgent requirement. In fact, since the 2009 election
campaign, the BJP had promised such an employment guarantee scheme.
A few positive components sustain the hope for a renewed focus on the urban centres. Even though not
remarkable at the macro-policy level, they focus more on delivery and implementation mechanisms – for
example, the Jal Jeevan Mission (Urban), and the much-needed Urban Swachh Bharat Mission 2.0.

Nevertheless, the absence of drastic policy measures that take on unemployment and environmental challenges
will continue to make urban centres more unequal and unsustainable.

125. What does the author imply by the usage of the phrase, “electoral challenges”?
(a) Budget is focused on fulfilling election promises.
(b) Budgetary funds may be used to fund state elections.
(c) Budget is targeted towards electoral appeasement.
(d) To make known the tribulations in states where elections are upcoming.

126. What does the author assume in making this statement “This will further dismantle the existing largely public
bus transport in favour of metro rail”. Answer in context of the passage.
(a) People prefer metro railways more because they are faster and, hence, public bus system is getting
dismantled.
(b) Entry of private sector means that metros are going to be revamped and would be more comfortable and
convenient to use.
(c) More funds will flow towards metros, and hence, they would offer greater facilities than public bus system,
hence, attracting more passenger traffic.
(d) Metro rail would put further dent on the crippling public bus transport.
Head Office: 127, Zone II, MP Nagar, Bhopal |+91-7676564400| https://www.toprankers.com Page 31 of 36
127. The author argues that the budget was silent on the smart cities initiative as it has become an embarrassment for
the government. Which of the following statements if true, weakens the author’s argument?
(a) Smart cities initiative is part of an already running scheme, and hence, it was not essential to make it a part
of the current budget.
(b) Smart cities initiative is a long-term initiative, and hence, the annual budget which looks at annual spending,
is not the correct way to approach the same.
(c) Budget talks about the sectors to which fresh funds would be allocated, funds were already allocated to the
smart city initiative last year, and hence, it was not pertinent to talk about the same in this budget.
(d) All of the above.

128. Which of the following is true about the passage?


(a) Budget aims at effecting a PPP partnership in order to boost the urban transport infrastructure.
(b) Migrant workmen have been given benefits such as working hostels in urban housing schemes, along with
tax concessions.
(c) Labour welfare measures focus on how to revive the reduced employment and stresses on job creation
mechanisms.
(d) The government in the budget session did not ensure a separate allocation for the delivery of housing options
to migrant workers.

129. Which of the following conclusions can be arrived at, based on the arguments from the given passage?
(a) The budget has not delivered on the points it should have delivered on.
(b) Urban unemployment has been the only budgetary focus in term of allocation of funds.
(c) Budget measures are more rhetoric than substance, especially with regard to urban development.
(d) The budget is a highlight on rhetorics, with nothing to give to the masses.

Passage(Q.130-Q.133): Forget the parched wasteland that is Brexit Britain in bleak midwinter. Let me take you
to a place where the political terrain is so radically different as to be idyllic. In this land, rows of cheap starter
homes neatly line the blossoming streets of new garden cities, and no twenty something frets about getting on to
the housing ladder. Not a solitary infant goes to bed hungry. And when you or your loved ones fall ill, a trusty
family doctor is on call from 8am to 8pm, all seven days of the week. Already I hear you rumbling like a famished
teen at Nando’s: where is this paradise? Sweden or, at the outside, Singapore? No, my friend: it is the UK in
2020. Or at least, it was supposed to be, because all of those treasures and so many more were promised to arrive
by this year by the serious, sober all-powerful who run the country.

130. Out of the following options, which one agrees with the crux of the passage?
(a) British politicians are accused of felony on numerous counts.
(b) The British political scenario is in a shambles.
(c) The promises made by the British politicians remain unfulfilled.
(d) The condition of Britain has vastly improved after the decision of Brexit was taken.

131. The most appropriate logical corollary to the passage is:


(a) The average British citizen had hoped a lot from international politicians.
(b) The year 2020 was the year of unfulfilled targets for the British people.
(c) 2020 has witnessed the realization of many a goal of Britons.
(d) Most of the promises that were given in 2020 were kept by the British politicians.

132. Out of the following options, which one strengthens the argument given in the passage?
(a) The average Briton is a happy person because all his/her needs are met now.
(b) The average Briton is a rebellious person who aspires freedom from all kinds of tyranny.
(c) The average Briton is a hypocrite because s/he does not practice what s/he preaches.
(d) The average Briton is an aggrieved person who is not at all happy about the unfulfilled promises.
Head Office: 127, Zone II, MP Nagar, Bhopal |+91-7676564400| https://www.toprankers.com Page 32 of 36
133. ‘Forget the parched wasteland that is Brexit Britain in bleak midwinter. Let me take you to a place where the
political terrain is so radically different as to be idyllic.’ The author through the lines
(a) Brings out the paradox in the 2020 British political scenario.
(b) Introduces a rosy picture of Britain.
(c) Reaches a conclusion based on the year 2020
(d) Highlight the plight of the British public in the year 2020.

134. If a meaningful word can be formed using the 3rd, 5th, 6th , 8th , 9th and 12th letters of the word
'BACHELORISMS', then what is the fourth letter from the right end of the word formed? If no such word can
be formed, then mark X as your answer while if more than one such word can be formed, then mark Z as your
answer.
(a) L (b) X (c) E (d) Z

Directions (Q.135): Study the following information carefully and answer the questions given below:
135. ‘P × Q’ means ‘P is brother of Q’.
‘P ÷ Q’ means ‘P is mother of Q’.
‘P - Q’ means ‘P is father of Q’.
‘P + Q’ means ‘P is sister of Q’.
Which of the following expressions represents ‘J is daughter of K’?
(a) J + M ÷ L - K (b) K - L × N + P (c) J - L × K ÷ O (d) K - L + J ÷ Q

Head Office: 127, Zone II, MP Nagar, Bhopal |+91-7676564400| https://www.toprankers.com Page 33 of 36
SECTION - E: QUANTITATIVE TECHNIQUES
Directions(Q.136-Q.140): Study the information carefully and answer the questions.
The Funnel chart shows the data of applicants applying and clearing Govt. exam in 2022. Some data is missing
which you have to find using the information given in the questions.

136. If 18000 candidates applied for Govt. exam in 2022, out of which 50% attempted pre exam, out of which only
1
5
𝒕𝒉 cleared mains exam. Find the number of candidates who did not get joining.
(a) 330 (b) 300 (c) 320 (d) 220

137. If 2700 candidates cleared mains exam and 180 candidates got joining, then find the ratio between the values
(X+Y) and (X-Y).
(a) 330 (b) 440 (c) 60 (d) Can’t be determined

138. At the end of the process totally 180 applicants got joining. 1800 females cleared mains exam and the ratio
between the male to female who cleared mains exam is 3:2. Find the number of candidates who attempted pre-
exam.
(a) 15000 (b) 30000 (c) 12500 (d) 25000

139. Total of 120000 applicants submitted their form, out of which only 60% attempts pre-exam and 100 applicants
got their joining after clearing the interview. Find the number of applicants who cleared the mains exam is how
much per cent of the applicants who cleared the pre-exam?
(a) 33% (b) 10% (c) 25% (d) 50%
1
140. If 4 of the applicants who cleared the interview got their joining, and 75000 applicants attempted the pre exam.
Then find the number of applicants who cleared mains exam.
(a) 8725 (b) 9500 (c) 9375 (d) 10000

Head Office: 127, Zone II, MP Nagar, Bhopal |+91-7676564400| https://www.toprankers.com Page 34 of 36
Directions(Q.141-Q.150): Study the following caselet and answer the questions.
There are 1200 people living in Ludhiana. There are equal number of male and female living in Ludhiana. But
the literate population is 20% less than male population. There are 1 500 people living in Patiala. But out of them
only 675 are literate. The number of males in Patiala is 150 more than the illiterate people. A total of 2005
females live in these three cities. The ratio of literate to illiterate people is 9/11 in Chandigarh. A total of 1600
people live in Chandigarh.

141. What is the percentage of males living in Chandigarh?


(a) 40 (b) 42 (c) 49 (d) 45

142. What is the difference between the number of males and females in Patiala?
(a) 420 (b) 450 (c) 460 (d) 520

143. What is the percentage of total number of literate people living in all the three cities?
(a) 43.6% (b) 42.6% (c) 47.1% (d) 52.9%

144. What is the difference between the total number of males and females living in all the three cities?
(a) 270 (b) 280 (c) 290 (d) 300

145. If 20% of the females from all the respective cities are illiterate, then how many males are literate in all the three
cities?
(a) 200 (b) 271 (c) 350 (d) 300

Direction (Q.146-Q.150): Study the Following graph carefully to answer the questions:
Per cent Profit Earned toy Two Companies Over the Years
(𝐼𝑛𝑐𝑜𝑚𝑒 − 𝐸𝑥𝑝𝑒𝑛𝑑𝑖𝑡𝑢𝑟𝑒) × 100
Percent profit =
𝐸𝑥𝑝𝑒𝑛𝑑𝑖𝑡𝑢𝑟𝑒
70

60 60
55
52
50 50 50
47.5
45
Percent profit

42.5
40 40
37.5
35 35
Company A
30
Company B
20

10

0
2005 2006 2007 2008 2009 2010
Year

146. If the income of company A in the year 2006 was Rs 6.425 lakhs, what was its expenditure in that year?
(a) 4.7 lakhs (b) 5.2 lakhs (c) 4.5 lakhs (d) 4.4 lakhs

Head Office: 127, Zone II, MP Nagar, Bhopal |+91-7676564400| https://www.toprankers.com Page 35 of 36
147. If the expenditure of company A in the year 2005 was 3.6 lakhs, what was the amount of profit earned by it in
that year?
(a) 2.52 lakhs (b) 2.46 lakhs (c) 1.44 lakhs (d) 1.31 lakhs

148. If the expenditure of Company A in 2009 was 4 times the expenditure in 2005, what is the ratio of Income of
Company A in 2005 compared to 2009?
(a) 24:7 (b) 7:24 (c) 29:7 (d) 7:30
149. If in the year 2009 incomes of both the companies A and B were the same, what was the respective ratio of their
expenditures in that year?
(a) 7:5 (b) 16:15 (c) 23:21 (d) Cannot be determined

150. If the income of Company B in 2009 was 4 times its income in 2008, what is the ratio of expenditure of Company
B in 2009 compared to 2008?
(a) 15:4 (b) 4:15 (c) 3:14 (d) 14:3

USE FOR ROUGH WORK

Head Office: 127, Zone II, MP Nagar, Bhopal |+91-7676564400| https://www.toprankers.com Page 36 of 36

You might also like